You are on page 1of 192

Question 1

Question text

Fracture neck of humerus may cause:


Select one:
a. Loss of power in interossi
b. Wrist drop
c. Weakness of pronator muscle
d. Sensory loss at the area overlying deltoid
e. Paralysis of abductor polices brevis

Question 2

Question text

Septic arthritis of infancy usually affects which of the following joints?


Select one:
a. Shoulder
b. Knee
c. Elbow
d. Hip
e. Wrist

Question 3

Question text

Which of the following is NOT true of intertrochanteric fracture of femur?


Select one:
a. Limb shortening
b. Internal fixation is preferred
c. Avascular necrosis of femoral head
d. Malunion
Question 4

Question text

A 32-year-old man complains of inability to hold a piece of paper between


his index and middle fingers. Which of the following nerves is likely to have
been injured?
Select one:
a. Ulnar nerve
b. Musculocutaneous nerve
c. Radial nerve
d. Posterior interosseous nerve
e. Median nerve

Question 5

Question text

Which is NOT true regarding compartment syndrome?


Select one:
a. Reperfusion injury is an established cause
b. Can be easily misdiagnosed as DVT
c. Loss of peripheral pulses is an early sign
d. Motor weakness suggest bad prognosis
e. Urgent fasciotomy is indicated if there is no improvement

Question 6

Question text

Osteoblastic bone secondaries commonly arise from cancer of:


Select one:
a. Breast
b. Lung
c. Adrenal
d. Prostate

Question 7

Question text

Club foot is:


Select one:
a. Always needs a surgery
b. Characteristically associated with breech presentation
c. Most commonly of the equino-valgus variety
d. A consequence of placenta praevia
e. Known as talipesequinovarus

Question 8

Question text

Anterior dislocation of shoulder may be complicated by:


Select one:
a. Brachial plexus injury
b. All of them
c. Tear of rotator cuff
d. Fracture head of humerus

Question 9

Question text

The name associated with joint neuropathy is that of:


Select one:
a. Cushing
b. Addison
c. Moon
d. Charcot
e. Osier
Question 10

Question text

Characteristic radiological appearances of OA include:


Select one:
a. Subchondral sclerosis
b. Codman’s triangle
c. Widening of the joint space
d. Subluxation
e. No bone formation

Question 11

Question text

Displaced fracture of the tibial eminence leads to what ligamentous


insufficiency?
Select one:
a. Posterolateral corner
b. Medial collateral
c. None
d. Posterior cruciate
e. Anterior cruciate

Question 12

Question text

Which of these structures does not pass posterior to the medial malleolus?
Select one:
a. Flexor hallucislongus tendon
b. Tibialis posterior tendon
c. Flexor digitorumlongus tendon
d. Posterior tibial artery
e. Saphenous vein

Question 13

Question text

What is the most common site of bone metastases?


Select one:
a. Hands/feet
b. Proximal femur
c. Spine
d. Proximal humerus
e. Pelvis

Question 14

Question text

Colles’ fracture is:


Select one:
a. A fracture of the clavicle
b. Common in elderly women
c. Fracture of the die scaphoid
d. A fracture of the ankle joint
e. A fracture of the head of the radius

Question 15

Question text

Bony prominences on which you kneel:


Select one:
a. Intercondylar eminences of tibia
b. Patellae
c. Femoral condyles
d. Tibial condyles
e. Tibialtuberosities

Question 16

Question text

Ostaomalacia is characterized by the following features except:


Select one:
a. Deficient protein metabolism
b. Skeletal deformities
c. Demineralization of the bones
d. Slow epiphyseal closure
e. X-ray Looser 's zones

Question 17

Question text

Which of the following structures does not pass through the carpal tunnel?
Select one:
a. Median nerve
b. Flexor carpi radialis tendon
c. Flexor pollicislongus tendon
d. Flexor digitorumsuperficialis tendons
e. Flexor digitorumprofundus tendon

Question 18

Question text

Death 3 days after pelvic fracture is most likely to be due to:


Select one:
a. Pulmonary embolism
b. Fat embolism
c. Respiratory distress
d. Hemorrhage

Question 19

Question text

A trigger finger is:


Select one:
a. an atrophic index finger in a median nerve palsy
b. due to stenosingtenovaginitis affecting one of the flexor tendons in the
palm
c. a component of syndactyly
d. an inflamed index finger
e. an essential feature of the carpal tunnel syndrome

Question 20

Question text

The sequestrum in X-ray appears:


Select one:
a. Isodense as surrounding bone
b. Light
c. Any of the above
d. Dense

Question 21

Question text

Most often open reduction of fracture is required in:


Select one:
a. Compound fracture
b. Closed fracture with nerve injury
c. Non union
d. Unsatisfactory closed reduction
e. Fracture in children

Question 22

Question text

Peak bone mass attainment in both men and women is most dependent on
which sex-steroid?
Select one:
a. Progesterone
b. Cortisol
c. Estrogen
d. Growth Hormone
e. Testosterone

Question 23

Question text

Which of the following is not a clinical of rickets?


Select one:
a. Enlarged epiphysis
b. Irritability
c. Short stature
d. Localized bone pain
e. Frontal bossing

Question 24

Question text

To avoid the sciatic nerve, an injection into the buttock is best given into:
Select one:
a. Upper and outer quadrant
b. Upper and inner quadrant
c. Lower and inner quadrant
d. Lower and outer quadrant
e. The junction of the four quadrants

Question 25

Question text

Bone destruction as a result of multiple myeloma is primarily caused by


which of the following cell types?
Select one:
a. Plasma cells
b. Macrophages
c. Myeloma cells
d. Osteoclasts
e. Pericytes

Question 26

Question text

A greenstick fracture:
Select one:
a. Occurs chiefly in the elderly.
b. Does not occur in children.
c. Is a fracture where part of the cortex is intact and partis crumpled or
cracked
d. Is a spiral fracture- of tubular bone

Question 27

Question text

A Baker’s cyst is:


Select one:
a. prepatellar bursa
b. An implantation dermoid cyst occurring in the palms of those who
work in a bakery
c. A synovial cyst of the wrists of those who knead bread
d. A synovial cyst of the popliteal fossa
e. A synovial cyst of the ankle

Question 28

Question text

The quadriceps femoris muscle:


Select one:
a. Extends the hip
b. Abducts the knee
c. Extends the knee
d. Rotates the knee
e. Flexes the knee

Question 29

Question text

Fracture femoral neck can be diagnosed from:


Select one:
a. External rotation
b. Abduction.
c. Limb shortening
d. Limb shortening + External rotation

Question 30

Question text

The most common osteolytic metastases in bones are derived from the:
Select one:
a. Kidney
b. Breast
c. Lung
d. Stomach
e. Prostate

Question 31

Question text

The most serious complication of supracondylar fracture of the humerus is:


Select one:
a. Volkmann’s ischaemic contracture
b. Myositis ossificans of the brachialis muscle
c. Cubitus valgus deformity
d. Non union and delayed union
e. Media nerve injury

Question 32

Question text

The most common injury following pelvic fracture is of:


Select one:
a. Rectum
b. Urethra
c. Vagina
d. Bladder

Question 33

Question text

The carpal bone most commonly fractured is:


Select one:
a. Hamate.
b. Capitate.
c. Triquetrum.
d. Scaphoid

Question 34

Question text

Injury to deep peroneal nerve would result in which of the following?


Select one:
a. Weakness of ankle plantarflexion
b. Weakness of hindfoot eversion
c. Weakness of great toe flexion
d. Weakness of hindfoot inversion
e. weakness of great toe extension

Question 35

Question text

Tears of the meniscus of the knee result from which of the following strain:
Select one:
a. Combined flexion and rotation
b. Rotation
c. Abduction
d. Adduction
e. Hyperextension

Question 36
Question text
The following are rotator cuff muscles except:
Select one:
a. Subscapularis
b. pectoralis minor
c. teres minor
d. infraspinatous
e. supraspinatous

Question 37

Question text

Which of the following muscles is not located in the thenar group of


muscles?
Select one:
a. Opponenspollicis
b. Abductor pollicisbrevis
c. Flexor pollicisbrevis
d. Adductor pollicis
e. Opponensdigitiminimi

Question 38

Question text

An absent ankle jerk indicates involvement of which of the following nerve


root ?
Select one:
a. L3
b. L4
c. L5
d. S1
e. L2

Question 39

Question text

Which of the following nerve injuries has the best prognosis?


Select one:
a. Wallerian degeneration
b. Neuropraxia
c. Neurotmesis
d. Axonotmesis

Question 40

Question text

What is the second most important aspect in the treatment of fractures of


long bones:
Select one:
a. Accurate anatomical reduction
b. Antibiotics
c. Adequate nutrition of patient
d. Immobilization
e. Restoration of bone alignment

Question 41

Question text

A patient who has sustained an open wound on the leg is bleeding profusely.
Before the patient arrives in the hospital the safest method to stop the
bleeding is:
Select one:
a. Elevation of leg
b. Local pressure on wound and elevation of leg
c. Ligation of bleeding vessel
d. Pressure over femoral artery in groin
e. Use of tourniquet
Question 42

Question text

Which joint is most commonly affected in osteoarthritis?


Select one:
a. Knee
b. First MCPJ
c. Hip
d. Ankle
e. Elbow

Question 43

Question text

Osteoid osteoma originates from:


Select one:
a. Medullary cavity
b. All of them
c. Periosteum
d. Cortex

Question 44

Question text

Complications of plastering of fractures are the following except:


Select one:
a. Peripheral neuritis
b. Allergic dermatitis
c. Delayed or malunion of fracture
d. Ischemia of the limb distal to the plaster
e. Venous congestion and venous thrombosis

Question 45

Question text

The most pain-sensitive structure in a joint is the:


Select one:
a. Synovial membrane
b. Articular cartilage
c. Joint capsule
d. Skin and subcutaneous tissues
e. Bone end

Question 46

Question text

Immobilization of fractures of long bones should include:


Select one:
a. Fractured bone only
b. Joint involved in the fracture
c. Distal joint
d. Proximal joint
e. Both proximal and distal joints

Question 47

Question text

Which of the following cells is solely responsible for the deposition


and mineralisation of bone matrix?
Select one:
a. Bone lining cells
b. Osteocytes
c. Osteoblasts
d. Chondrocytes
e. Osteoclasts

Question 48

Question text

Which of the following is the best way to preserve amputated parts for
replantation?
Select one:
a. Dry cooling with ice
b. Immersion in cold antibiotic solution
c. Deep freezing
d. Immersion in cold ringer lactate
e. Immersion in cold saline

Question 49

Question text

An 18-month-old girl presents to your office for an initial health supervision


visit. The mother notes that the girl limps, and she thinks one leg is
longer than the other. There is no history of medical attention for this
problem in the adoption documentation. Review of available medical history
and the current physical examination reveal no signs of illness. The child
appears to be otherwise growing and developing normally. On physical
examination, you measure a leg length discrepancy of 2 cm, and the girl has
a “waddling” type limp and difficulty in abducting her right thigh at the hip.
Of the following, the MOST likely diagnosis is:
Select one:
a. developmental dysplasia of the hip
b. Legg-Calvé-Perthes disease
c. slipped capital femoral epiphysis
d. femoral anteversion (internal femoral torsion)
e. vitamin D-dependent rickets

Question 50

Question text

What is the earliest indication of Volkmann’s ischaemia:


Select one:
a. Pain
b. Paraesthesia in median nerve area
c. Pallor and poor capillary filling
d. Contracture of fingers
e. Gangrene of tips of fingers

1) Origin of bone is from:

a. Ectoderm.
b. Mesoderm.
c. Endoderm.
d. All of the above.

2) Acute osteomyelitis is commonly caused by:

a. Staph aureus.
b. S. pyogenes.
c. H. influenzae.
d. Salmonella.

3) Acute osteomyelitis usually begins at:

a. Epiphysis.
b. Metaphysis.
c. Diaphysis.
d. Any of the above.

4) What is not True of acute pyogenic osteomyelitis:

a. Trauma is a predisposing factor.


b. Common infecting agent is Staph. Aureus.
c. Infection is usually blood borne.
d. All are true.

5) What is not True of Brodie's abscess:

a. A form of chronic osteomyelitis.


b. Intermittent pain and swelling.
c. Common to diaphysis.
d. Excision is very often required.
6) Tuberculosis of the spine most likely originates from:

a. Intervertebral disk.

b. Cancellous vertebral body.

c. Ligamentous structures.

d. Paravertebral soft tissue.

7) In Pott's spine, the disease starts in the:

a. Intervertebral disk.

b. Anterior vertebral margin.

c. Posterior vertebral margin.

d. Paravertebral soft tissue.

8) Melon seed bodies in joint fluid are characteristic of:

a. Rheumatoid arthritis.

b. Tuberculous arthritis.

c. Septic arthritis.

d. None of the above.

9) The earliest sign of TB hip in X-ray is:

a. Narrow joint space.

b. Irregular moth eaten femoral head.

c. Periarticular osteoporosis.

d. Dislocation.

10) Healing of tuberculous arthritis can lead to:

a. Calcification.

b. Fibrous ankylosis.

c. Boney ankylosis.

d. None of the above.


11) Osteoid osteoma originates from:

a. Periosteum.

b. Cortex.

c. Medullary cavity.

d. All of the above.

12) Sun ray appearance of osteosarcoma is because of:

a. Periosteal reaction.

b. Osteonecrosis.

c. Calcification along vessels.

d. None of the above.

13) Bone metastasis in male commonly arises from

cancer of:

a. Lung.

b. Prostate.

c. Kidney.

d. Thyroid.

14) Osteoblastic bone secondaries commonly arise from

cancer of:

a. Breast.

b. Lung.

c. Prostate.

d. Adrenal.

15) Bone metastasis can be best evaluated by:

a. X-ray.

b. 99mTC bone scan.

c. 111Indium scan.
d. Calcium-alkaline phosphatase elevation.

16) The synonym for Paget's disease is:

a. Osteitis fibrosa.

b. Osteitis proliferans.

c. Osteitis deformans.

d. None of the above.

17) Multiple myeloma tumor cells resemble:

a. Granulocytes.

b. Plasma cells.

c. Lymphocytes.

d. Chondrocytes.

18) An adamantinoma historically contains:

a. Squamous cell rests.

b. Pallisading cells.

c. Cells resembling basilar cells.

d. All of the above.

19) Osteomalacia predominantly affects the:

a. Spine.

b. Pelvis.

c. Skull bones.

d. Metatarsals.

20) The enzyme found in osteoclasts but not in osteoblasts

is:

a. Alkaline phosphatase.

b. Acid phosphatase.

c. Elastase.
d. Cytochrome oxidase.

21) A Gigli saw is:

a. An electrically driven circular bone saw.

b. A pneumatically driven bone saw.

c. A short straight bone saw.

d. A long twisted wire bone saw.

22) Osteoclasis can be used to:

a. Correct deformity of the tibia due to rickets.

b. Curette an osteoclastoma.

c. Correct deformity.

d. Correct a ricketery rosary.

23) In Dupuytren's contracture which one of the following statements incorrect:

a. It is a contracture of the flexor tendons to the ring and little fingers.

b. It is a contracture of the palmar fascia.

c. It may occur in the plantar fascia.

d. There is an association with cirrhosis of the liver.

24) In an adult patient with a fracture of the shaft of the

femur:

a. No blood can be lost without obvious swelling.

b. No blood can be lost without obvious bruising.

c. Two liters of blood can be lost without obvious swelling or bruising.

d. There is no possibility of death from hemorrhagic shock.

25) A greenstick fracture:

a. Occurs chiefly in the elderly.

b. Does not occur in children.


c. Is a spiral fracture- of tubular bone.

d. Is a fracture where part of the cortex is intact and part is crumpled or cracked.

26) Spiral fracture is due to:

a. Blunt trauma.

b. Axial compression.

c. Twist.

d. Direct impact.

27) The single most important factor in fracture healing is:

a. Correct bone alignment.

b. Accurate reduction.

c. Immobilization.

d. Organization of clot.

28) Immobilization is not required in fracture involving:

a. Scapula.

b. Wings of ilium.

c. Rib.

d. Proximal humerus in elderly.

e. All of the above.

29) Internal reduction is considered in presence of:

a. Reduction impossible to the achieved or maintained.

b. Healing is expected to be delayed.

c. Pathological fracture.

d. All of the above.

30) Which one of these statements is True in diagnosis

of congenital hip dislocation in the first few days of life:

a. It is impossible to diagnose it.


b. The sign of telescoping is the best way of diagnosing it.

c. It is possible to diagnose it by the Van Rosen/Barlow Test.

d. The Trendelenberg test is the most useful.

31) Trendelenburg's sign is used in the diagnosis of:

a. Varicose veins.

b. Congenital dislocation of the hip.

c. Carcinoma of the stomach.

d. Pulmonary embolism.

32) If an unstable hip is detected at birth the management policy is:

a. Do nothing and re-examine every six months as only a minority of hips develop into a
persistent dislocation.

b. Use a splint to keep the hip joint in 45° flexion and adduction.

c. Use a splint to keep the hip joint in 90° flexion and abduction.

d. Advise operative stabilization.

33) The essential examination of the hip in order to clinch the diagnosis of chronic slipped
femoral epiphysis is:

a. Measuring for shortening of the leg.

b. Palpation of the femoral head.

c. A-P plain x-ray view of the hip.

d. Lateral x-ray view of the hip.

34) Shenton's line is a sign applicable to:

a. The detection of shortening of the leg on physical examination.

b. A radiological feature of the pelvis applied to the diagnosis of congenital dislocation of the
hip.

c. A radiological feature of the lungs applied to the diagnosis of pulmonary vein thrombosis.
d. A physical sign applied to the diagnosis of adrenal deficiency.

35) Perthes' disease is common to age group of:

a. 1-5.

b. 6-10.

c. 11-15.

d. 16-20.

36) The average duration of Perthes' disease is:

a. 1-2 years.

b. 3- 4 years.

c. 1 month - 6 months.

d. 6 months - 1 year.

37) In Perthes' disease the hip movements restricted are:

a. Abduction and external rotation.

b. Abduction and internal rotation.

c. Adduction and external rotation.

d. All of the above.

38) The sequestrum in X-ray appears:

a. Dense.

b. Light.

c. Isodense as surrounding bone.

d. Any of the above.

39) The term delayed union is employed when the fracture fails to unite within:

a. 1.5 times the normal union time.

b. Twice the normal union time.

c. 2.5 times the normal union time.

d. None of the above.


40) First bone to ossify in foetal life is:

a. Femur.

b. Tibia.

c. Clavicle.

d. Sternum.

41) What is True of clavicle fracture:

a. Non-union is rare.

b. Malunion is of no functional significance.

c. Reduction even if achieved is difficult to maintain.

d. All are true.

42) The joint most likely to have recurrent dislocation is:

a. Ankle.

b. Knee.

c. Shoulder.

d. Patella.

43) Anterior dislocation of shoulder may be complicated by:

a. Brachial plexus injury.

b. Tear of rotator cuff.

c. Fracture head of humerus.

d. All of the above.

44) What is not True about fracture surgical neck of humerus:

a. Occurs due to fall on outstretched hand.

b. Common to children.

c. Osteoporosis is an important risk factor.

d. Non-union is uncommon.

45) Radial nerve palsy may occur in fr of humerus involving:


a. Surgical neck.

b. Shaft.

c. Lower end.

d. At all of the above locations.

46) The most common form of supracondylar fracture humerus in children is of which type:

a. Flexion.

b. Extension.

c. Combination of A & B

d. None of the above.

47) Myositis ossificans commonly occurs around:

a. Shoulder.

b. Elbow.

c. Wrist.

d. Knee.

48) Volkman's ischemia commonly occurs following:

a. Fracture shaft humerus.

b. Supracondylar fracture.

c. Colles' fracture.

d. Monteggia fracture.

49) The single dependable sign of early Volkmann's contracture is:

a. Cyanosis of fingers.

b. Obliteration of radial pulse.

c. Paralysis of flexor muscles of forearm.

d. Pallor of fingers.
e. Pain.

50) Cubitus valgus of elbow commonly follows fracture of:

a. Lateral condyle.

b. Medial condyle.

c. Capitalum.

d. Lower third of humerus.

51) Fracture involving which part of humerus can cause

delayed ulnar palsy:

a. Shaft.

b. Surgical neck.

c. Medial epicondyle.

d. Lateral epieondyle.

52) The deformity of wrist in Colles' fracture is:

a. Madelung's deformity.

b. Dinner fork deformity.

c. Buttonaire deformity.

d. None of the above.

53) Colles' fracture can be complicated by late rupture of:

a. Extensor pollicis longus.

b. Abductor pollicis longus.

c. Adductor pollicis longus.

d. Flexor pollicis longus.

54) What is True of Sudeck's atrophy of hand:

a. Hand is painful and swollen.

b. Osteoporosis of carpals and metacarpals.

c. There is increased blood flow to para-articular areas.


d. Cervical sympathectomy may be of help.

e. All are true.

55) The carpal bone most commonly fractured is:

a. Triquetrum.

b. Hamate.

c. Capitate.

d. Scaphoid.

56) Which nerve is compressed in carpal tunnel syndrome:

a. Ulnar.

b. Median.

c. Radial.

d. All of the above.

57) The most common injury following pelvic fracture is of:

a. Bladder.

b. Urethra.

c. Rectum.

d. Vagina.

58) Limb shortening with adduction and internal rotation occurs in which type of hip
dislocation:

a. Anterior.

b. Posterior.

c. Central.

d. All of the above.

59) Nelaton's line joins anterior superior iliac spine to:

a. Xiphisternum.

b. Pubic tubercle.

c. Ischial tuberosity.
d. Ischial spine.

60) Bryant's triangle helps to assess:

a. Fracture neck of femur.

b. Iliac crest displacement.

c. Trochanteric displacement.

d. None of the above.

61) Normal neck-shaft angle of femur is:

a. 90°.

b. 120°.

c. 150°.

d. 170°.

62) Fracture femoral neck can be diagnosed from:

a. Limb shortening.

b. External rotation.

c. Abduction.

d. A + B.

63) Which of the following is not True of intertrochanteric fracture of femur:

a. Limb shortening.

b. Malunion.

c. Avascular necrosis of femoral head.

d. Internal fixation is preferred.

64) Most common complication of fracture shaft femur is:

a. Malunion.

b. Nonunion.

c. Knee stiffness.

d. Fat embolism.

65) Spontaneous bleeding into joints in haemophilia occurs when factor VI level is less than:
a. 50%.

b. 25%.

c. 10%.

d. 5%.

66) Recurrence of Baker's cyst should make the surgeon suspect:

a. Neoplastic change.

b. Undiagnosed pathology within knee.

c. Incomplete removal of the cyst.

d. The communication to the joint is persisting.

67) Flexion of distal interphalangeal joint with fixing the proximal interphalangeal joint (PIP)
tests:

a. Flexor digitorum profundus.

b. Flexor digitorum superficials.

c. Palmaris longus.

d. All of the above.

68) Stenosing tenovaginitis commonly affects:

a. Abductor pollicis.

b. Flexor pollicis longus.

c. Opponens pollicis.

d. All of the above.

69) A sequestrum is:

a. a piece of soft dead tissue

b. a piece of dead skin

c. a dead tooth

d. a piece of dead bone


e. a retained swab

70) Union of a simple uncomplicated transverse fracture of the tibia in an adult normally
takes

a. 6 weeks

b. 8 weeks

c. 12 weeks

d. 18 weeks

e. 26 weeks

71) Fractures which do not impact include:

a. fracture of tile vault of the skull

b. a compression fracture

c. a simple fracture

d. a transverse fracture of the patella

e. fracture of the neck of the femur

72) Colles' fracture is

a. a fracture of the clavicle

b. a fracture about the ankle joint

c. common in elderly women

d. a fracture of the head of the radius

e. fracture of die scaphoid

73) Bennett's fracture is:

a. reversed Colles' fracture

b. fracture of the scaphoid bone in the wrist

c. fracture of the radial styloid (chauffeur's fracture)

d. fracture dislocation of the first metacarpal

e. cause of mallet finger.


74) Supracondylar fracture of the humerus in a child:

a. is due to a fall on the point of the elbow

b. is usually compound

c. requires admission of the patient after reduction

d. requires immediate open reduction

e. is a fracture dislocation

75) A fracture of the midshaft of the clavicle is best treated by:

a. clavicle rings

b. a figure-of-eight bandage

c. open reduction and plating

d. an intramedullary nail

e. a broad arm sling and analgesics

76) A Pott's fracture is a type of fracture of the:

a. wrist

b. ankle

c. spine

d. foot

e. skull

77) Treatment of a severe comminuted fracture of the patella includes:

a. physiotherapy alone

b. insertion of a figure-of-eight tension band

c. patellectomy

d. inserting screws or wire

e. skin traction

78) Malunion of a fracture is:

a. a fracture which unites in a position of deformity


b. delayed union of a fracture

c. non-union of a fracture

d. followed by pseudoarthrosis

e. due to tuberculosis

79) Volkmann's contracture:

a. affects the palmar fascia

b. develops at the ankle in a case of chronic venous ulcer

c. follows ischemia of the forearm muscles

d. is due to excessive scarring of the skin of the axilla following a burn

e. follows ulnar nerve palsy

80) A Brodie's abscess is:

a. a subperiosteal abscess due to infection of the mastoid air cells

b. a type pf breast abscess

c. a chronic abscess of the bone

d. an abscess arising in the inguinal lymph nodes

e. an abscess forming in an infected varicose vein

81) The initial abnormality in primary osteoarthritis is:

a. in the synovial membrane

b. sclerosis of cartilage

c. fibrillation of cartilage

d. an osteophyte

e. a pannus

82) The initial abnormality in rheumatoid arthritis is:


a. fibrillation of cartilage

b. sclerosis of cartilage

c. in the synovial membrane

d. in the capsule

e. proliferation of bone

83) Pott's paraplegia is due to:

a. hematomyelia following trauma

b. damage to the cord by a piece of bone when vertebrae collapse in tuberculosis of the
spine

c. tuberculous pus and angulation in tuberculosis of the spine

d. damage to die corda equina after a fall

e. fracture dislocation of cervical vertebrae

84) Still's disease is:

a. spastic diplegia

b. rheumatoid arthritis in childhood

c. rheumatoid arthritis in the elderly

d. post-traumatic bone formation in the lateral ligament of the knee

e. synonymous with Reiter's disease

85) A benign tumor forming osteoid is:

a. a synovioma

b. a chondroma

c. an osteoma

d. a fibroma

e. an adenoma
86) Ewing's tumor affecting the humerus

a. is a metastasis from carcinoma of the thyroid

b. should be treated by immediate amputation

c. looks like a cut onion on x-ray

d. has a soap-bubble appearance on x-ray

e. displays sun-ray spicules on x-ray

87) Barlow's sign is related to the diagnosis of:

a. talipes equinus varus

b. congenital dislocation of the hip

c. ulnar nerve palsy

d. genu varum

e. fractured neck of femur

88) The reported incidence of unstable hips per 1000 at birth is as much as:

a. 0.5

b. 2-5

c. 8-20

d. 25-30

e. 35-40

89) The word talipes refers to:

a. long feet with spidery toes

b. knock knee

c. flat feet

d. hammer toes

e. club feet
90) Bone dysplasia is due strictly to:

a. faulty nutrition

b. osteomyelitis

c. parathyroid tumor

d. trauma

e. faulty development

91) Idiopathic scoliosis is a:

a. lateral curvature of the spine

b. rotation of the spine

c. lateral curvature with rotation of the spine

d. flexion deformity of the spine

e. congenital disease with hemivertebrae

92) A Milwaukee brace can be used in

a. sacro-iliac strain

b. derangement of the teeth

c. a patient with an above knee amputation

d. scoliosis

e. fractured clavicle

93) Legg-Calve-Perthe's disease is:

a. osteochondritis of the spine

b. tuberculosis of the hip joint

c. slipped proximal femoral epiphysis

d. osteochondritis of the proximal femoral epiphysis

e. osteomalacia
94) The name associated with joint neuropathy is that of:

a. Cushing

b. Osier

c. Moon

d. Charcot

e. Addison

95) Adrenocorticosteroids administered in excess cause:

a. osteoporosis

b. osteosclerosis

c. osteochondritis

d. endochondral ossification

e. osteosarcoma

96) 'Tennis elbow' is the term used for:

a. olecranon bursitis

b. non-articular rheumatism' of the extensor muscles of forearm attached to lateral


epicondyle of the humerus

c. myositis ossificans of the supinator muscle

d. a fractured head of radius

e. ulnar nerve neuritis

97) An adventitious bursa is:

a. an anatomical bursa overlying any joint

b. a type of degeneration of adventitia of popliteal artery

c. an acquired bursa generated from connective tissue

d. a pseudocyst in the lesser sac (omental bursa)

e. an infected knee
98) A trigger finger is:

a. an inflamed index finger

b. an atrophic index finger in a median nerve palsy

c. due to stenosing tenovaginitis affecting one of the flexor tendons in the palm

d. an essential feature of the carpal tunnel syndrome

e. a component of syndactyly

99) A Baker's cyst is:

a. an implantation dermoid cyst occurring in the palms of those who work in a bakery

b. a synovial cyst of the wrists of those who knead bread

c. a prepatellar bursa

d. a synovial cyst of the ankle

e. a synovial cyst of the popliteal fossa

100) Immobilization of fractures of long bones should include:

a. Fractured bone only

b. Joint involved in the fracture

c. Proximal joint

d. Both proximal and distal joints

e. Distal joint

101) Non-union is common in fractures of the following bones except the:

a. Carpal scaphoid

b. Neck of the femur

c. Lower third of the tibia

d. Talus

e. Tuberosity of the fifth metatarsal


102) The signs of fractured shaft of a bone do not include:

a. Swelling

b. Deformity

c. Loss of all movements in the limb

d. Acute localized bone tenderness

e. Abnormal mobility in the line of the bone

103) In the following types of fractures of long bones, crepitus can be elicited only in:

a. Fissures

b. Subperiosteal cracks

c. Greenstick fractures

d. Spiral and oblique fractures

e. Impacted fractures

104) The most severe growth disturbance results from which of the following types of
epiphyseal injuries:

a. Separation of the epiphysis at the metaphyseal side of the epiphyseal plate

b. Separation of the epiphysis with a triangular fragment of the metaphysic

c. Intra-articular fracture involving the articular cartilage epiphysis and epiphyseal plate

d. Intra-articular fracture extending from the joint surface through the epiphysis and
epiphyseal plate to the metaphysic

e. Crashing injuries compressing the epiphyseal plate without displacement

105) Local complications of closed fractures do not

include:

a. Malunion

b. Non-union

c. Infection
d. Sudek's atrophy

e. Joint stiffness

106) Non-union in closed fractures may due to any of the following except:

a. Inadequate immobilization

b. Interposition of soft parts

c. Impaired blood supply

d. Inpsction of the fragments

e. Wide separation of the fragments

107) Causes of gangrene after fracture in a limb do not include:

a. Direct crushing of the tissues

b. Injury to the main vessels

c. Tight plasters

d. Septic infection

e. Clostridial infection

108) The correct ttt of traumatic myositis ossificans is by:

a. Prolonged immobilization

b. Active exercises

c. Passive stretching and massage

d. Both A and B

e. Both B and C

109) Concerning fracture of the shaft of the clavicle, it is untrue that it:

a. Is usually due to direct trauma

b. Commonly involves the middle third


c. Is often associated with overriding of fragments

d. Causes dropping and deformity of shoulder

e. Is usually treated by figure-of-eight bandage


broad arm sling
and analgesics

110) A child with midclavicular fracture and overriding of the fragments is best treated by:

a. Supine bed rest with interscapsular sandbag support

b. Open reduction and internal fixation

c. Figure-of-eight bandage

d. Closed reduction and plaster fixation

e. Manipulative reduction and abduction splint

111) In shoulder dislocations, the humeral head usually dislocates primarily in which of the
following directions:

a. Inferiorly

b. Superiorly

c. Anteriorly

d. Posteriorly

e. Laterally

112) The incorrect statement about anterior dislocation of the shoulder joint is that:

a. shoulder loses its rounded contour & becomes flattened

b. The elbow is abducted from the side

c. All movements of the shoulder are limited and painful

d. The anterior and posterior folds of the axilla are elevated

e. The hand cannot be placed on the opposite shoulder (Duga's test)

113) Recent dislocations of shoulder joint are best treated by:

a. Hippocrates' method of closed reduction


b. Kocher's manipulation

c. Modified Milch's manoevre

d. Open reduction

e. Putti-platt1s operation

114) Recurrent shoulder joint dislocation is best treated by:

a. Physiotherapy

b. Nicola's operation

c. Bankart's operation

d. Putti-Platt's operation

e. Arthrodesis of the joint

115) Fractures of the shaft of the humerus are best treated by:

a. Closed reduction and shoulder spica

b. Continuous skeletal traction

c. Open reduction and internal fixation

d. Hanging plaster cast

e. Coaptation plaster splint with a Velpeau dressing

116) The most vulnerable structure in supracondylar fracture of the humerus is the:

a. Median cubital vein

b. Brachial artery

c. Median nerve

d. Ulnar nerve

e. Radial nerve

117) Posterior dislocation of elbow joint is characterized by the following except:


a. Gross swellinq of the elbow region

b. Loss of all movements at the elbow joint

c. Shortening of the upper arm

d. Absence of crepitus

e. Loss of the normal relationship of the olecranon with the two eoicondvles

118) In fracture of the olecranon process of the ulna, the following statements are true
except that it:

a. Is usually due to a fall on the elbow

b. Can be felt as a gap between the olecranon and the shaft

c. Is rarely associated with hemarthrosis

d. May be complicated by anterior dislocation of the elbow joint

e. Always requires surgical treatment

119) Concerning extension Monoteggia's fracturedislocation, it is untrue that it:

a. Consists of fracture of the upper third of the ulna and anterior dislocation of the radial
head

b. Is usually due to a severe blow on the back of the forearm

c. Can be treated by manipulative reduction in children

d. Always requires surgical treatment in adults

e. Is rarely associated with complications

120) An elderly ♀ sustained Colles' fracture which was properly treated. However, she
developed severe pain & stiffness of the wrist with coldness and cyanosis of the hand. X-ray
examination revealed diffuse decalcification of the bones. She proved to be suffering from:

a. Causalgia

b. Tuberculous arthritis of wrist joint

c. Traumatic tenosynovitis

d. Sudek's atrophy
e. Osteoarthritis of wrist joint

121) Following a stumble on stairs, a 70-year-old ♂ felt severe pain in the hip and could not
stand up. O/E, there was shortening of the limb, external rotation deformity and tender
thickening of the greater trochanter. X-ray examination revealed:

a. Intracapsular fracture of the neck of the femur

b. Pertochanteric fracture of the femur

c. Dislocation of hip

d. Fracture of acetabulum

e. Fracture of greater trochanter

122) Tears of the meniscus of the knee result from which of the following strain:

a. Hyperextension

b. Abduction

c. Adduction

d. Rotation

e. Combined flexion and rotation

123) In a football game, an athlete felt severe pain in his Rt knee while turning to the left
side with the joint flexed and taking the body weight. Soon after, the joint became swollen
and painful but recovery followed rest for 3 weeks. Thereafter, the patient suffered from
recurrent locking with pain and a feeling of "giving way" in the joint. The most probable
diagnosis is:

a. Solitary loose body

b. Fracture of the tibial spine

c. Rupture of the medial ligament

d. Rupture of the medial semillunar cartilage

e. Fracture of the patella

124) A lateral blow at the level of the knee joint may cause:
a. Rupture of anterior cruciate ligament

b. Rupture of medial collateral ligament

c. Avulsion of medial meniscus

d. Bumper fracture of tibia

e. All of the above

125) A march fracture most frequently results from:

a. Direct trauma

b. Jumping from a height

c. Muscle fatigue from prolonged walking

d. Use of high-healed shoes

e. Osteoporosis

126) Intestinal absorption of calcium is dependent upon:

a. Vitamin D

b. Parathoraone

c. Calcitonin

d. All of the above

e. None of the above

127) An irregular epiphyseal line with calcifying periosteal haematc found on X-ray
examination is indicative of:

a. Infantile rickets

b. Scurvy

c. Hemophilia

d. Hypoparathyroidism

e. Hypervitaminosis A
128) Osteoporosis is a deficiency in:

a. Calcium metabolism

b. Calcium deposition

c. Protein supporting tissue

d. All of the above

e. None of the above

129) Which of the following statements is untrue concerning the enzyme alkaline
phosphatase:

a. Has a normal serum concentration of 3-13 KA units

b. Is present in high concentrations in liver cells

c. Is excreted in the bile

d. Is elevated in the serum of patients with healing fractures

e. Is ↑ in the serum of patients with rickets and osteomalacia

130) Localized bone sclerosis may be due to:

a. Syphilis

b. Sclerosing osteoperiostitis

c. Osteoarthritis

d. Bone tumors

e. All of the above

131) Enlarged tender epiphyses with bowing of long bones and X-ray evidence of delayed
carpal ossification suggest the diagnosis of:

a. Scurvy

b. Infantile rickets

c. Syphilitis epiphysitis

d. Osteogenesis imperfecta
e. Achondroplasia

132) Osteomalacia may be due to the following factors except:

a. Starvation

b. Repeated pregnancies

c. Idiopathic steatorrhea

d. Increased renal excretion of calcium and phosphorus

e. Prolonged recumbency

133) Achondroplasia is characterized by the following features except:

a. Short extremities

b. Normal trunk length

c. Normal intelligence

d. Enlarged head

e. Normal ossification of cartilage

134) Osteogenesis imperfecta is characterized by the following features except:

a. Blue sclerae

b. Brittle shell-like bones

c. Multiple fractures

d. Osteoporosis

e. Familial tendency

135) Concerning Ollier's disease, which of the following statements is incorrect:

a. There is a strong hereditary predisposition

b. Multiple enchondromata occur in small long bones of hands and feet

c. The affected bones are stunted


d. Deformities may arise from unequal affection of metaphyses

e. Chonrosarcoma may develop in one of the enchonromata

136) The following statements about diaphyseal aclasis are true except that it:

a. Is a common hereditary condition

b. Never affects membrane bones

c. Is characterized by multiple exostoses

d. May be associated with dwarfism

e. Spares the metaphyses of long bones

137) Concerning Osgood-Schlatter's disease, the following statements are true except that
it:

a. Is an avascular necrosis of the epiphysis of the tibia tubercle

b. Usually occurs between ages of 10 and 16 years

c. Is due to traction of the patellar tendon on the tibial tubercle

d. Is associated with no radiological signs

e. Causes pain and swelling over the tubercle

138) Generalized osteitis fibrosa cystica is characterized by the following features except:

a. Deficiency of parathormone

b. Diffuse decalcification and softening of bones

c. Cysts containing Brwonish fluid

d. Multiple gaint-cell tumors

e. Urinary symptoms due to renal calculi

139) The following statements about Paget's disease of bones are true except that it:

a. Affects elderly subjects, particularly males

b. Is a generalized bone dystrophy of obscure etiology


c. Causes deformities in the skull, spine, pelvis and lower limbs

d. Produces no pressure symptoms

e. Is often associated with cardiovascular complications

140) Which of the following statements is untrue in Paget's disease:

a. The bones are greatly thickened and very vascular

b. Cranial nerve palsies may occur

c. Spontaneous fractures are rare

d. X-ray examination of the skull is diagnostic

e. Osteogenic sarcoma occurs in over 5% of cases

141) Which statement is untrue in renal rickets:

a. Results from renal insufficiency in infancy

b. Is due to deficient phosphorus, excretion

c. Manifests itself by marked dwarfism

d. Causes no deformities in the limbs

e. Ends fatally from uraemia at puberty

142) A 7-year-old child presented with intermittent limp and pain in the right hip and knee.
On examination, flexion and extension movements were free and there was no tenderness
and no muscle wasting. X-ray examination confirmed the diagnosis of:

a. Early tuberculous arthritis of the hip joint

b. Traumatic arthritis

c. Perthes' disease

d. Slipped upper femoral epiphysis

e. Coxa vara
143) A 9-year-old boy presented with limping and pain in the right knee two days after a fall
in the street. On examination he looked ill and in severe pain with high fever and swelling of
the knee region extending to the thigh which was warm and very tender. The most probable
diagnosis is:

a. Traumatic synovitis

b. Hemarthrosis

c. Acute osteomyelitis of the femur

d. Septic arthritis of knee

e. Bone sarcoma

144) Solitary bone cyst is characterized by the following features except that it:

a. Occurs most often in children and adolescents

b. Usually arises in the diaphysis of a long bone

c. Often remains symptomless until complicated by pathological fracture

d. Appears as clear ovoid expanding cavity in the X-ray

e. May be associated with new-bone formation

145) The most common tumor of the small bones of the hands and feet is:

a. Enchondroma

b. Osteochondroma

c. Osteoclastoma

d. Cancellous osteoma

e. Bone sarcoma

146) Which of the following is most common in the small bones of the hands and feet:

a. Osteochondroma

b. Enchondroma

c. Osteoid osteoma

d. Osteochondritis juvinelis

e. Tuberculous osteitis
147) Concerning osteoclastoma, the following statements are correct except that it:

a. Usually occurs between the ages of 15 and 40 years

b. Always arises in metaphyseal region of cartilaginous bones

c. Consists of large giant cells in a very vascular stroma of spindle cells

d. Presents as a painless globular swelling with welldefined edge

e. Produces diagnostic radiological signs

148) The radiological signs of osteoclastoma include the following except:

a. Abrupt expansion of the bone

b. Characteristic soap-bubble appearance

c. Presence of an operculum obliterating the medullary cavity

d. Absence of any new-bone formation

e. Presence of areas of bone destruction

149) The treatment of osteoclastoma includes the following measures except:

a. Curettage of tumor tissue & packing cavity with bone chips

b. Excision with safety margin of bone

c. Amputation

d. Radiotherapy

e. Chemotherapy

150) The following statements about multiple myeloma are true except that it::

a. Is a primary malignant tumor of bone marrow

b. Occurs between the ages of 40 and 60 years

c. Usually presents with bone pain especially in the back

d. Is rarely associated with fever and anemia


e. May cause paraplegia with girdle pains

151) The following statements about bone sarcoma are true except that it:

a. Arises from osteoblasts of the periosteum or bone cortex

b. Forms a fusiform mass ensheathing the bone

c. Often invades the epiphyseal cartilage and neighbouring joint

d. Produces characteristic new bone formation in the X-ray

e. Disseminates rapidly by the blood stream

152) Which one of the following statements is untrue concerning chondro-sarcoma:

a. Occurs most often between the ages 20 and 60 yrs

b. Is always a primary malignant tumor of bone

c. Most commonly affects scapula, pelvis, ribs & sternum

d. Causes bone expansion and destruction with irregular opacities in the X-ray

e. Is radioresistant

153) Ewing's sarcoma is characterized by the following except that it:

a. Is a common tumor of children

b. Always arises in the metaphysis of a long bone

c. Presents as a fusiform swelling with inflammatory changes in the overlying soft tissues

d. May be associated with leucocytosis

e. Produces characteristic radiological signs

154) The most important DD of Ewing's tumor is:

a. Chondrosarcoma

b. Osteogenic sarcoma

c. Acute osteomyelitis
d. Malignant metastasis

e. Reticulum cell sarcoma

155) The most common osteolytic metastases in bones are derived from the:

a. Lung

b. Breast

c. Stomach

d. Kidney

e. Prostate

156) The most pain-sensitive structure in a joint is the:

a. Bone end

b. Articular cartilage

c. Joint capsule

d. Synovial membrane

e. Skin and subcutaneous tissues

157) Ostaomalacia is characterized by the following features except:

a. Deficient protein metabolism

b. Demineralization of the bones

c. Slow epiphyseal closure

d. Skeletal deformities

e. X-ray Looser 's zones

158) The following statements about solitary bone cyst are true except that it:

a. Occurs most often in children and adolescents

b. Is commonest in the humerus , femur and tibia

c. Arises in the diaphysis of the bone


d. Assumes an avoid shape and may cause bone expansion

e. May remain innoticed until complicated by pathological fracture

159) The differential diagnosis of osteoid osteoma includes all the following except:

a. Brodies' abscess

b. Ossifying fibroma

c. Ewing's tumor

d. Metastatic thyroid nodule

e. Bone sarcoma

160) Ivory osteomata occur most often in the:

a. Skull

b. Spine

c. Humerus

d. Femur

e. Tibia

161) Sensory end organs are absent from the:

a. Muscle

b. Periostium

c. Synovial membrane

d. Articular cartilage

e. Joint capsule

162) The viscosity of synovial fluid is mainly due to:

a. Chondroitin sulphate

b. Hyaluronidase

c. Albumin and laucin

d. Alpha and beta globulins


e. Beta lipoproteins

163) Septic arthritis of infancy usually affects which of the following joints:

a. Shoulder

b. Elbow

c. Wrist

d. Hip

e. Knee

164) The treatment of acute septic synovitis includes the following except:

a. Massive antibiotics

b. Splintage in the position of function

c. Aspiration and antibiotic injection

d. Arthrotony and drainage

e. Excision and Winnett Orr-treatment

165) Rheumatoid arthritis primarily involves the:

a. Articular cartilage

b. Subchondral bone

c. Synovial membrane

d. Capsule

e. Ligaments

166) Complications of rheumatoid arthritis in the hands include:

a. Tenosynovitis

b. Rupture of extensor tendons

c. Carpal tunnel syndrome

d. Ulnar deviation at the metacarpophalangeal joints

e. Bony ankylosis of affected joints


167) A 20-year-old male presented because of increasing pain in his left lower thigh.
Examination revealed tender fusiform thickening of the lower end of the femur with a small
effusion into the knee joint. The overlying skin was warm and the seat of dilated veins but
movements of the knee were free and painless. X-ray examination revealed:

a. Acute osteomyelitis of the lower end of the femur

b. Brodie's abscess

c. Bone sarcoma

d. Parosteal fibrosarcoma

e. Ewing's tumor

168) Bloody or coffee-ground fluid obtained by aspiration of the knee joint is suggestive of:

a. Septic arthritis

b. Hemophilic joint

c. Synovial chondromatosis

d. Pigmented villonodular synovitis

e. Charcot's joint

169) Correct statements regarding the carpal tunnel syndrome include the following except
that it:

a. Is always due to compression of the median nerve in

the carpal tunnel

b. May follow a wrist fracture

c. May occur in patients with rheumatoid arthritis

d. Frequently first appears during pregnancy

e. Is often associated with vascular disorders

170) A 60-year-old male with 3 months history of severe back-ache anemia and loss of
weight, developed severe girdle pains with weakness of the lower limbs. Examination
revealed low grade fever with marked tenderness over the spine, ribs, sternum, skull and
pelvic bones. X-ray examination of the skeleton revealed multiple punched out defects
without any new bone formation. The most probable diagnosis is:
a. Bone metastases from an occult primary

b. Multiple myeloma

c. Osteitis fibrosa cystica

d. Hand Schuller-Christian's disease

e. Paget's disease

171) 12-year-old boy developed bilateral painless effusion of both knees together with
blurring of vision and impairment of hearing. He should be suspected to be suffering1 from:

a. Traumatic synovitis.

b. Tuberculous arthritis.

c. Inherited syphylis.

d. Rheumatic arthritis.

e. Rheumatoid arthritis.

172) A 40-year-old male presented with a grossly swollen painless left knee. Examination
revealed a flail joint with irregularly thickened bone ends, palpably swollen synovial
membrane and marked grating and creaking on passive movement of the joint. The first
diagnostic step is:

a. Examination of the nervous system.

b. Serological tests.

c. X-ray examination of the joint.

d. Examination of aspirated synovial fluid.

e. Arthroscopy and synovial biopsy.

173) A 9-year-old boy developed an intermittent limp which soon became constant and
associated with pain in the Rt hip & knee. Examination revealed a flexion deformity of the Rt
hip with limitation of flexion and extension movements, wasting of the thigh muscles and
upward tilting of the pelvis. The most probable diagnosis is :

a. Congenital dislocation of the hip.

b. Legg-Perthes' disease.
c. Septic arthritis of infancy.

d. Tuberculous arthritis.

e. Slipped upper femoral epiphysis.

174) The X-ray findings in tuberculosis of the hip include the following except :

a. Diffuse decalcification of the bones.

b. Blurring of the joint outline.

c. Diminution of the joint space.

d. Wandering acetabulum.

e. Downward tilting of the pelvis.

175) In children, the treatment of tuberculosis of the hip includes the following except :

a. Tuberculostatic drugs.

b. Weight traction to correct deformity.

c. Fixation of the joint in the position of function.

d. Aspiration of cold abscess.

e. Extra-articular arthrodesis.

176) In Sprengel's shoulder, the following statements are correct except that :

a. There is congenital elevation & maldevelopment of scapula.

b. An ugly prominence in the neck is produced by the superior angle.

c. A band of fibrocartilage or bone anchors the medial border of the scapula to the spine.

d. There is no limitation of shoulder movements.

e. No ttt is required apart from excision of the supermedial angle of scapula to improve the
appearance.

177) The following statements about cubitus valgusm deformity are correct except that :

a. It may be due to malunited supracondylar fr of the humerus or non united fr of the lateral
condyle
b. The deformity is most obvious when elbow is fully flexed

c. It predisposes to delayed ulnar neuritis

d. Treatment of supracondylar osteotomy is necessary only when the deformity is severe

178) A 25 years old male complained of limitation of wrist movements, especially


dorsiflexion, and weakness of the hand, especially the grip. Examination revealed radial
deviation of the hand with abnormal prominence of the ulna. He gave a history of a fall on
the outstretched hand in children. The correct diagnosis is :

a. Rheumatoid arthritis of the wrist joint.

b. Non-united fracture of the scaphoid.

c. Malunited Colles' fracture.

d. Madelung's deformity.

e. Persistent dislocation of the lunate.

179) The most characteristic feature of Volkmann's contracture is :

a. Wasting of the forearm.

b. Flexion deformity of the wrist.

c. Extension of the metacarprophalangeal joints with flexion of the interphalangeal joints.

d. Vokamann's phenomenon.

e. Weakness of the hand and fingers.

180) The following statements about Dupuytren's contracture are true except that :

a. It is due to contraction of the palmar fascia which starts as an indurated nodule on the
ulnar border of the hand.

b. The ring and little fingers are severely affected.

c. The deformity consists of flexion of the metacarprophalangeal and proximal


interphalangeal joints with extension of the terminal joint.

d. The skin overlying the indurated fascia is often puckered and immobile.

e. The joint capsules and flexor tendons are not affected.


181) The most effective ttt of sever Dupuytren's contracture is:

a. Repeated stretching and night splintage.

b. Local injections of fibrinolysin or hydrocortisone.

c. X-ray therapy.

d. Subcutaneous fasciotomy.

e. Radical excision of the palmar fascia.

182) The most diagnostic sign of congenital hip dysplasia in the newly borne is :

a. Widening of the perineum.

b. Asymmetry of the buttocks.

c. Ortalani's sign.

d. Limitation of hip abduction with hip and knees flexed to 90°.

e. Apparent shortening of the thigh with the hips and knees flexed to 90°.

183) The earliest radiological sign in congenital hip dislocation in infants is :

a. The small shallow acetabulum.

b. The hypoplastic femoral head.

c. The shortened anteverted femoral neck.

d. Distortion of Shenton's line.

e. Displacement of the femoral head from the acetabulum.

184) In congenital dislocation of the hip (CDH), the pathological changes include the
following except :

a. Small shallow acetabulum.

b. Snail flattened femoral head lying outside the acetabulum.

c. Elongated femoral neck.

d. Thickened adherent joint capsule with an hour-glass constriction.

e. Shortened hamstrings and adductors.


185) Trenderburg' s sign can be elicited in all of the following except :

a. Congenital dislocation of the hip.

b. Infantile paralysis of the gluteal muscles.

c. Coxa vara.

d. Tuberculous arthritis of the hip joint.

e. Non-united fracture of the femoral neck.

186) A 10-year-old male with neglected congenital dislocation of the hip presented because
of increasing pain in the back with limping and fatigue. The appropriate management should
be :

a. Analgesics and anti-inflammatory drugs.

b. Raising the heel of the right shoe.

c. Open reduction with deepening the acetabulum by a shelf procedure.

d. Colonna's arthroplasty.

e. Lorenz's bifurcation osteotomy.

187) The clinical features of coxa vara include the following except :

a. Shortening, addution and eversion of the limb.

b. Raising of the greater trochanter above Nelaton's line.

c. Limitation of all movements of the hip.

d. Positive Trendelenburg's sign.

e. Limping and difficulty in kneeling, riding and separating the legs.

188) Treatment of genu valgum includes the following except:

a. Physiotherapy.

b. Wedged shoes.

c. Night splints.

d. Osteoclasis of the tibia.

e. McEwen's osteotomy of the femur.


189) A 4-year-old rachitic child developed severe genu varum. The best line of treatment is
by :

a. Antirachitic treatment.

b. Repeated moulding.

c. Corrective splints.

d. Osteoclasis of the tibia.

e. Tibial osteotomy.

190) A 13-year-old boy presented for consultation because his knees tend to knock together
and he tends to fall during running. Examination revealed separation of the medial malleoli
by 3 inches when standing with the knees in contact with each other and the feet directed
forwards. The correct diagnosis is :

a. Coxa vara.

b. Genu valgum.

c. Genu varum.

d. Talipes calcaneovalgus.

e. Bilateral flat foot.

191) The most common congenital deformity of the hindfoot is talipes :

a. Calcaneus.

b. Equinus.

c. Eguinovarus.

d. Varus.

e. Valgus.

192) Paralytic talipes is differentiated from, congenital talipes by the following features
except that :

a. The deformity appears later after birth.

b. The limb is atrophied, cyanosed and cold.

c. The muscles are wasted and flabby.


d. Usually both sides are affected.

e. The deformity can be corrected easily by manipulation.

193) An 8-year-old child with neglected congenital equinovarus is best treated by :

a. Repeated manipulation under anaesthesia.

b. Denis-Browne splint.

c. Plantar fasciotomy and elongation of the tendo Achilles.

d. Wedge tarsectomy.

e. Dunn's triple arthrodesis.

194) The commonest form of acquired talipes is the :

a. Paralytic.

b. Spastic.

c. Traumatic.

d. Cicatricial.

e. Compensatory.

195) The best treatment for pes cavus causing severe local pressure on the metatarsal-heads
is by :

a. Toe exercises.

b. Electric stimulation of the intrinsic muscles.

c. Steindler's operation.

d. Lambrinudi's operation.

e. Dunn's triple arthrodesis.

196) The commonest variety of flat foot is the :

a. Congenital.

b. Spasmodic.

c. Paralytic.

d. Statis.
e. Traumatic.

197) A 25-year-old male suffered from painful swelling of the feet and ankles over the last 6
weeks. On examination, the arches were preserved but the skin was congested and localized
tenderness could be elicited over the navicular bone and the spring, deltoid and plantar
ligaments. The correct diagnosis is :

a. Sprain of the ankle joint.

b. Plantar fasciitis.

c. Incipient flat foot.

d. Spasmodic flat foot.

e. Talonavicular arthritis.

198) An adolescent male complained of severe pain in the foot and leg after prolonged
standing. Examination revealed that the foot is flat and fixed in extreme eversion by spastic
contraction of the peroneal muscles and long extensors of the toes. The most likely
diagnosis is :

a. Incipient flat foot.

b. Spasmodic flat foot.

c. Tuberculosis of the ankle joint.

d. Sprain of the ankle.

e. Retrocalcanean bursitis.

199) Which of the following is associated with neurofibromatosis ?

a. Talipes equinovarus.

b. Metatarsus varus.

c. Pseudarthrosis of the tibia.

d. Genu recurvatum.

e. Congenital hip dysplasia.

200) Regarding hallux valgus, the following statements are true except that it:
a. Consists of outward deviation of the great toe at the metatarso-phalangeal joint.

b. Is usually due to badly fitting shoes.

c. Is not progressive.

d. Causes hammer-toe deformity in the other toes.

e. Predisposes to several painful complications.

201) The causes of hammer-toe include the following except:

a. Overcrowding of the toes by ill-fitting shoes.

b. Hallux valgus.

c. Pes cavus.

d. Talipes equinus.

e. Rupture of the extensor expansion.

202) The following statements about provisional amputation for infective gangrene are true
except that it:

a. May be urgently needed to control infection and toxemia.

b. Should be made through the healthy limb above the infected area.

c. Should be done as low as possible to allow reamputation at the optimum level.

d. Should provide free drainage.

e. May be carried out by the guillotine or flap method without closure.

203) The ideal amputation should fulfill the following requirements except that it should:

a. Be as long as possible.

b. Have a smoothly rounded cone-shaped end.

c. Not include muscle over the bone end.

d. Have a linear freely movable scar not exposed to pressure.

e. Be painless with a freely movable joint above and a smooth bone end elbow.

\
204) Syme's amputation is better than a below-knee amputation except that it:

a. Is less "catastrophic" to the patient.

b. Allows the patient to walk around in his room without prosthesis.

c. Maintains, the pleasure of "earth feeding".

d. Requires a cheap stump boot.

e. Is not attended with serious complications.

205) Reimplantation of a traumatically amputated limb requires all of the following except:

a. Limb preservation.

b. Shortening of bone.

c. Immediate arterial and venous repair.

d. Routine angiograms.

e. Delayed repair of nerves.


1. A fracture is said to be:

a. closed if an overlying skin laceration has been sutured

b. simple when there is a single fracture line

c. comminuted if there has been associated damage to

adjacent nerves or vessels

d. a fatigue fracture if it occurs through a diseased bone

e. pathological if it occurs through a bony metastasis

2. In a healing fracture:

a. the hematoma is initially invaded by osteoblasts

b. the tissue formed by the invading osteoblasts is termed osteoid

c. osteoid tissue is formed in an acid pH

d. calcium salts are laid down in the osteoid tissue

e. the final stage of repair is the remodeling of the callus

3. Non-union is often seen in:

a. fractures of the fourth metatarsal

b. fractures of the neck of the femur

c. fractures of the condyle of the mandible

d. Colles’ fractures

e. scaphoid fractures

4. Dislocation of the sternoclavicular joint:

a. is usually caused by a fall on the outstretched hand

b. displaces the clavicle upwards and medially

c. is usually treated by internal fixation

d. very rarely causes any compression of the trachea or vessels in the neck
e. is usually accompanied by fracture of the first rib

5. Fractures of the clavicle

a. Are usually of the greenstick variety in children under the age

b. are usually the result of direct violence

c. are frequently associated with injury to the subclavian vessels

d. can be recognized by the abnormal elevation of the fragment

e. are usually treated by internal fixation

6. Fractures of the neck of the scapula

a. are often due to a fall on the outstretched hand

b. are frequently associated with chest wall injury

c. are often associated with dislocation of the acromioclavicular

d. can usually be managed without surgical intervention

e. are often associated with fracture of the corocoid process

7. Recurrent dislocation of the shoulder

a. is usually in the posterior position

b. is usually in young adults

c. is more common after associated damage to the glenoid labrum

d. usually requires surgical repair

e. is surgically managed by tightening the soft tissues over the inferior aspect of the joint

8. In fractures of the surgical neck of the humerus the:

a. lesion is usually due to indirect violence

b. fracture line usually passes between the greater and lesser tuberosities

c. fragments are usually impacted


d. proximal fragment Is usually internally rotated

e. distal fragment is usually adducted

9. In a fracture of the distal 1/3 of the shaft of the humerus:

a. the distal fragment is usually posteriorly angulated by the action of biceps

b. the radial nerve is frequently damaged

c. delayed radial nerve palsy is usually due to edema

d. late onset of radial nerve palsy is usually due to the involvement of the nerve with callus

e. ulnar nerve palsy is usually of late onset

10. A supracondylar fracture of the humerus

a. is a fracture commonly seen in young adults

b. is particularly subject to the complication of ischemic muscle contracture

c. is held in the position of reduction by the tendon of brachioradialis

d. when properly reduced has the index finger pointing approximately to the tip of shoulder
of the same side

e. is commonly accompanied by ulnar nerve palsy

11. Fractures of the head of the radius

a. do not occur in isolation

b. are usually associated with dislocation of the radius

c. may be associated with dislocation of the elbow joint

d. may require surgical excision of the head

e. are usually accompanied by damage to the median nerve

12. In a Monteggia fracture dislocation

a. the dislocation of the distal radio-ulnar joint brings the ulnar styloid process anterior to
the capitulum
b. the radial fracture is usually at the junction of the middle and distal thirds

c. internal fixation is usually required in the adult

d. the causative injury is often a blow on the extensor surface of the forearm with the elbow
flexed

e. the commonest neurological injury is to the posterior branch of the radial nerve

13. In a Colles' fracture the distal radial fragment

a. is dorsally angulated on the proximal radius

b. is usually torn from the intra-articular triangular disc

c. is deviated to the ulnar side

d. is usually impacted

e. commonly damages the median nerve

14. Fractures of the radial styloid

a. extend into the wrist joint

b. typically have an anterior dislocation of the bony fragment

c. are commonly associated with fractures of the triquetral bone

d. are commonly associated with fractures of scaphoid

e. are commonly associated with dislocation of the wrist joint

15. A transverse fracture of the scaphoid is:

a. the commonest carpal injury

b. prone to infection

c. usually seen in young men

d. prone to avascular necrosis

e. usually seen on an early scaphoid radiograph

16. In pelvic fractures:


a. avulsion injuries are usually treated by early mobilization

b. undisplaced lesions of the ischial or pubic rami are usually treated by early mobilization

c. extraperitoneal urinary extravasation may be due to damage either to the membranous


urethra or to the base of the bladder

d. extraperitoneal urinary extravasation may be due to damage of the base of the bladder

e. which are unstable, one half of the pelvis is displaced proximally by the flank muscles

f. Reduction may need 40 to 50 Ib (18 to 23 kg) of traction


1. A greenstick fracture:
a.Occurs chiefly in the elderly.
B. Does not occur in children.
C.Is a spiral fracture of tubular bone.
D. fracture where part of the cortex is intact and part is crumpled or cracked.

2. As regards Non-union all are correct except:

A. Diagnosed if there is no healing after 6 months.


B. Characterized by thinning of bone ends in avascular nonunion.
C. be treated conservatively in avascular nonunion.
D. May result from excess union trials.
E. Is rare in fractures of highly vascularized bones, e.g. clavicle.

3. As regard open fracture all are correct except :

A. Gustilo et al classification is useful in determining method of fixation.


B. Wound debridement means removal of foreign bodiesand devitalized tissues.
C. Closure of skin wounds is always indicated.
D. non contratcing non bleeding muscles should be excised

4. Delayed union:

A. May be caused by infection .


B. Systemic steroids may cause delayed union.
C. Is diagnosed when there is nonunion after 1.5 times as the expected time for
union.
d. is treated by treatment of the cause + rigid fixation.
e. All of the above

5. Which is true concerning fracture healing:

A. Delayed union refers to fracture healing which takes longer than normal.
B. Non- union refers to complete of a fracture to unite.
C. Mal union refers to a fracture which unites but in anon-anatomical position.
D. A pathological fracture refers to a fracture which occurs inbone weakened by
pre- existing disease .
E. All of the above

6. Which statement is untrue regarding Sudek's atrophy:

A. Occurs most common after wrist and ankle injuries.


B. Is characterized by severe pain and stiffness .
C.Followed by osteoarthritis of the near joints .
D. May require sympathectomy .
E. Is a type of osteodystrophy .

7. The most common complication in both Pott's and Colle's fracture is:

A. Malunion.
B. Sudek's atrophy.
C. Neurovascular bundle.
D.Osteoarthritis.

8. The local complications of closed fractures do not include:

A. Malunion.
B. Non-union.
C. Infection
D. Sudek's atrophy.
E. Joint stiffness.

9. In the following types of fractures of bones, crepitus can be elicited only in :

A. Fissures.
B. Subperiosteal cracks.
C. Greenstick fractures
D. Spiral and oblique fractures
E. Impacted fractures

10. Regarding Volkmann's ischemic contracture, the following statement are correct,
EXCEPT:

A. It is a reversible condition .
B. It is a grave complication of supracondylar fracture humerus.
C. It can be avoided by early fasciotomy.
D. It leads to permanent disability to the patient
e. it involves the flexors of the wrist and fingers .

11. After trauma, the following are " definitive" signs of fracture, EXCEPT:

A. Swelling with ecchymosis and tenderness .


B. Shortening.
C. Abnormal movements associated with severe pain
D. Crepitus.
E. Deformity of the limb.
12. The following are characteristic of fractures in children, EXCEPT:

A. Green stick fractures may occur


B.Fractures take a longer time to heal than in adults.
C.Physeal injuries may cause limb deformity.
D. Fracture remodeling is more than in adults.
E. Rehabilitation is much easier than in adults

13. Indications for internal fixation include the following EXCEPT:

A. Oper fractures
B. Vascular injuries
C. Polytraura with multiple fractures
D. Pathological fractures

14. Myositis ossificans commonly occurs around:

A. Shoulder
B. Elbow
C. Wrist
D. Knee

15. Which of the following statements regarding compartment syndrome FALSE ?

A. It is detected by a loss of distal pulses and sensation


B. It produces pain out of all proportion to the injury.
c. is more common in closed fractures than open ones.
D. It is result of raised pressure in a compartment collapsing the veins.
E. It is treated by a fasciotomy

16 .A 60-year- old man is a frontseat passenger in a car crash. He is found to have


three fractured ribs on the right rupture of the liver, pelvic fracture, right femoral
fracture, and a left tibial fracture. The patient is given broad- spectrum antibiotics,
and his injuries are managed surgery, requiring 12 of blood . The patient improves
initially , but on the third postoperative day , he develops hypoxia (PuO2 , 55 mm Hg
), with confusion, tachypnes, and petechia . What the most likely diagnosis

A. Recurrent intra -abdominal hemorrhage from dilutionalthrombocytopenia


B. Transfusion reaction
C. Antibiotic allergy
D. Fat embolism
e. Disseminated intravascular coagulopathy (DIC)
17. Non union in closed fractures may be due to any of the following EXCEPT :
A inadequate immobilization ,
B. Soft tissue interposition .
c.Impaired blood supply.
D. Impaction of the fragments
E Wide separation of the fragments.

18. Concerning fracture of the shaft of the clavicle, it is untrue that it:

A. Is usually due to direct trauma.


B. Commonly involves the middle third.
C. Is often associated with overriding of fragments.
D. Causes dropping and deformity of shoulder.
E. usually treated by figure-of-eight bandage.

19. The most common site of fracture clavicle is:


A. Middle.
B. Medial end
c. Between middle and lateral thirds.
D. Between middle and medial thirds .

20. Most common complication of fracture clavicle is :

A. Brachial plexus injury.


B. Shoulder stiffness.
c. Malunion (of no functional significance ).
d. of the above

21. In dislocation shoulder, Which of the following is the commonest:

a. Subclaviular dislocation .
B. Subacromial dislocation
C. Subspinous dislocation.
D. Subcoracoid dislocation

22. The treatment of recurrent shoulder distocation (The commonest


complication of anterior shoulder dislocation)is:

A. Kocher's method
B. Hippocratic method.
C. Bankart's operation .
D. Arthrodesis
23. Alf of the following are common complications of anterior shoulder dislocation
except:
A .Rotator tear.
B. Radial nerve injury.
C. Axillary nerve injury.
D. Brachial plexus injury.
E. Recurrent dislocation.

24. The incorrect statement about anterior dislocation of the shoulder joint is:

A. The shoulder loses its round contour and become flattened.


B. The elbow is abducted from the sides.
C. All movements of the shoulder are limited and painful.
D. The anterior and posterior folds of the axilla elevated.
E. The hand cannot efevated on the opposite shoulder.

25.Regarding fracture surgical neck of the Humerus all are true, Except:

a. May be due to a fall on outstretched hand.


b.In unimpacted fracture, upper fragment is adducted and lower fragment is
abducted.
C. Results injury of axillary nerve.
D. Treatment is by closed reduction

26. Radial nerve paralysis is most often associated with:

A. Fracture of the surgical neck of the humerus.


B. Spiral fracture of the humerus.
c.Supracondylar fracture of the humerus.
D. Fracture of the lateral condyle of the humerus.
e. Fracture of the radial head

27. Closed fracture the shaft of the humerus is best treated by

A. Closed reduction and shoulder spica .


B. Intra medullary nall .
C. Plate and screws fixation
D. U- shaped plaster cast and arm to neck
E. Skeletal traction

28. A 7- year -old boy off bicycle , tanding on the left elbow . He presents to the
emergency room with massive , swelling of the elbow with paintul and restricted
elbow motion rays show a displaced fracture of the distal end of the humerus .
Which the following is the most serious complication of this fracture ?
a. Nonunion of fracture fragments
B. Nonunion of fracture Tragments with
C. Disruption of the growth plate at the distal end of the humerus
D. Forearm compartment syndrome (Volkmann's ischemia )
E. Ankylosis of the elbow joint

29. The following fractures are more common in postmenapausal females except :

A .Fracture surgical neck of humerus .


B. Fracture neck femur .
C. Supracondylar humerus .
D. Colle's fracture .

30. The following are presentation of supracondylar fracture except :

A. Disturbance of supracondylar ridge


B. Disturbance of equidistant triangle between medial & lateral epicondyles and
condylar process of olecranon .
C. Partial limitation around elbow .
D. Injury of neurovascular bundle .

31. The most vulnerable structure in supracondylar fracture of the humerus is the :

A. Median cubital vein .


B. Brachial artery
c. Median nerve .
D. Ulnar nerve E. Radial nerve .

32. Vascular injury is famous with this fractures:

a. Neck humerus .
B. Both bones upper limb .
C. Pelvis .
d. Supracondylar femur

33. Colle's fracture is :

A. Extraarticular fracture of distal uina .


B. Intraarticular fracture of distal ulna .
C. Extraarticular fracture of distal radius .
D. Intraarticular fracture of distal radius .

34. All of the following are possible complications of Colle's fracture except:
A. Sudek's atrophy .
B. Radial artery Injury
C. Carpal tunnel syndrome .
D. Rapture of extensor policislongus tendon.
E. Myositis ossificans

35. A 72 year old woman fell on outstretched armThis was foflowed by pain and by
swelling above the wrist. Plain X-showed fracture of the distal radius. About this
fracture, all the following statements are true , except :

A. This is called Colles' fracture .


B. There is anterior displacement and angulation of distal fragment
C. There is radial displacement and angulation of distal fragment
D. Osteoporosis is the usual predisposing factor .
E. The majority of these cases are treated by closed reduction, and immobilization
in a plaster cast.

36. You are asked to see a patient in the outpatients clinic. The patient has
weakness in her left arm following a car accident 1 month ago when she fractured
her left arm. On examination there is weakness of extension of the fingers and
wrist on the left side. However, sensation is maintained in all distributions and
there is no wrist drop. Which one of the following fractures classically associated
with nerve damage is the most likely cause of this palsy?

A . Fracture of head of radius


B. Fracture of shaft of humerus
c. Medial epicondyle of humerus
D. Fracture of shaft of ulna
E.Fracture of neck of femur

37. As regards fracture of scaphold bone all are correct except:

A. common carpal bone fracture


B. May not affect function and may not appear in early -, film
C. Avascular necrosis doesn't affect function.
D. Is an intra-articular fracture .

38. The most consistent sign of fracture of carpal of scaphoid is:


a. wrist pain during attempted push ups
b. diffuse swelling on the dorsum of wrist
c. localized tenderness in the anatomic snuffbox
d. wrist popping on movement
39. Concerning extension Monoteggia's fracture -dislocation, it is untrue that
it:

A. Consists of fracture of the upper third of the ulna and anterior dislocation
of the radial head .
B. Is usually due to a severe blow on the back of the foream.
C. Can be treated by manipulative reduction in children
D. Always requires surgical treatment in adults
E. is rarely associated with complications

40. In a man with a traumatic pelvic disruption, blood at the uretheral meatus
is best investigated by:

a. Execretory IVU

b. Urinanalysis.

C. Uretheral catheterization.

D. Retrograde uretherogram.

41. The commonest complication of fracture pelvis is:

A. Injury to penile urethera.

B. Injury to buibomembranousurethera.

C. Scrotal injury.

D. Injury to the rectum.

e. Injury to the bladder

42. A 40-year-old man is hit by a car and sustains an injury to the pelvis. Which
of the follow most indicative of a urethral injury?

A. Hematuria

B. Scrotal ecchymosis

C. Oliguria

D. High-riding prostate on rectal examination

E. Intravenous pyelography ( IVP) showing dye extravasation in the pelvis


43. The following fractures affect hip joint stability except:

A. Butterfly fracture

b. Fracture of sacrum.

C. Malgaigne fracture.

D. Open book fracture

44. The following are complications of hip fracture except:

A. Sciatic nerve injury.

B. Obstetrical difficulties .

C. Hemorrhagic shock

D. Complications of prolonged recumbency .

E. Femoral nerve injury.

45. Open book fracture means :

a. Separation of sacroiliac joints on both sides .

B. Separation of sacroiliac joint on one side with double pelvis fracture .

C. Wide separation of symphysis pubis with little separation of sacroiliac joint


on one side .

D. Total pelvic disruption

46. Regarding joint dislocations , only one of the following is false :

a. The most common joint to dislocate is the shoulder .

b. The most common joint dislocation lo be complicated by recurrence is the


shoulder.

C. Hip dislocation is usually anterior .

d. Shoulder dislocation is usually anterior .

E. With joint dislocation there is complete loss of joint movement .


47. The most common type of traumatic hip dislocation:

A .Anterior

B. Posterior.

C.Inferior

D.Central

48. The earliest radiological sign in congenital hip dislocation in infants is:

A .The small shallow acetabulum .

b. The hypoplastic femoral head

C. The shortened anteverted femoral neck.

D. Distortion of Shenton's line.

E. Displacement of the femoral head from the acetabulum.

49. Complications of posterior hip dislocation include the following except:

A. Obturator nerve injury.

B. Sciatic nerve injury.

C. Irreducibility

D. Complications of prolonged recumbency .

50. All of the following are methods for reduction of hip dislocation except:

A. Open reduction if associated fracture acetabulum preventing closed reduction.

B. Kocher's method

C. Allis method.

D. Stimson method.

51. Position of lower limb in posterior dislocation is:

A .Flexed abducted with shortening.


B. Flexed adducted with shortening.

C. Flexed abducted with lengthening

D. Flexed adducted with lengthening.

52. Positive obturator sign:

A. Injury of obturator nerve.

B. Disruption of obturator ring

C. Medial displacement of radiographic obturator line formed by fat over obturatorinternus.

D. Tear of obturatorexternus

53. Arthroplasty defined as:

A. Replacement of the articular surface of a long bone by synthetic material

B. Correction of angulation of long bones

c.Open reduction and internal fixation of intra- articular fracture

D. Open reduction and internal fixation of long bone fracture

E. Open drainage of an infected joint

54. The most important complication of fracture neck femur is :

a. Septic shock.

B. Ischemic necrosis(AVN)

c. Fat embolism.

D. Bed sores.

55. Mortality rate in the 1st 3 months in fracture neck femur is:

A. 5%.

B. 10 %
C. 20%

D. 35%

56. Complications of prolonged recumbency in bed is famous for this fracture:

A. Neck femur.

B. Supracondylar femur.

C. Shaft tibia.

D. Pott's fracture

57. An otherwise fit 64 year old patient is admitted after fall at home, -ray showed non
displaced intracapsular neck of femur, the appropriate management is:

A. Total hip replacement

B. Cannulated screw fixation

c.Hemiarthroplasty

D. Dynamic hip screw fixation

58. The usual position of unimpacted fracture neck femur is:

A. External rotation and adduction.

B. Internal rotation and adduction.

C. External rotation and abduction.

D. d. Internal rotation and adduction

59. The more possibly injured nerve in fracture neck femur is:

A. Femoral nerve.

B. Obturator nerve.

C. Sciatic nerve.

D. None of the above


60. In extra capsular fracture neck femur, the used method of fixation will be:

A. External fixation.

B. Total hip replacement

C. Dynamic hip screw (DHS)

D. Austin Moore prosthesis

61. About complications of fractures, all the following statements are true, except:

A. Fracture of the shaft of humerus can cause wrist drop.

B. Fracture of the medial epicondyle of humerus can cause claw hand deformity.

C. Fracture of the surgical neck of humerus can cause atrophy of deltoid muscle.

D. Extra- capsular fracture of neck of femur can cause necrosis of its head.

E. Supracondylar fracture of the femur can cause popliteal artery injury.

62. A 35 year old male was hit by a car and injured his left femur. X-rays revealed a
comminuted fracture of the mid shaft of the left femur. The following statements are
correct, EXCEPT:

A. Patient should be evaluated to exclude associated hip /pelvic injury .

B. Patient should be evaluated to exclude associated visceral injury.

C. Patient should be evaluated to exclude hypovolemic shock.

D. Ideal treatment is fixation in hip spica cast till healing .

E. Ideal treatment is internal fixation by closed interlocking intramedullary nail.

63. A transverse fracture of the shaft of the femur in 20 years male is best treated by:

A. Intra-medullary nail

B. Skin traction.

c. Skeletal traction

D. Plate and screws fixation


64. Regarding treatment of fracture shaft of femur, all are true Except:

a. Children up to 5 years by gallow's traction

B. In adolescents by skin traction on Thomas' splint

C. Transverse fracture in adults by intramedullary nail

D. Compound fracture in adults by interlocking nails

65. As regards treatment of fracture shaft femur all are correct except:

A. Newborn with mid shaft fracture Crede's method.

B. Comminuted supracondylar external fixation.

C. Subtrochanteric fracture condylar plate or interlocking nail.

D. Adult ORIF

66. A 30 year old male patient received a direct blow to the knee during a fight. He
presented to the emergency room with loss of active extension of the knee. X- ray was done
showing a transverse fracture of the patella. The management should be:

A. Rest.

B. Above knee plaster cast.

C. Internal fixation.

D. Patellectomy .

67. A 25-year-old man experiences pain in the right knee while skiing, causing his knee to
twist and him to fall to the ground. His knee is swollen. He cannot bear full weight or fully
extend or bend his leg. There is tenderness over the medial joint line. Emergency room x-ray
findings were normal, and the range of motion (ROM), although restricted, is stable to varus
and valgus stress. Straight-leg raise is unrestricted. Which is the most likely type of injury?

A. Anterior cruciate ligament

B. Tuberosity

C. Transverse genicular ligament

D. Medial meniscus
E. Posterior cruciate ligament

68. A 35 year old patient presented to the emergency room after a road traffic accident with
a compound fracture of both tibia and fibula of the left leg, the leg is tense and tender with
no felt pedal pulsations . The first thing to do for this patient is:

A. Fasciotomy

B. External fixation

C. Internal fixation

D. Exploration of the leg vessels

69. A 15-year-old athlete with knee pain and severe tenderness at the tibial tubercle most
likely has:

A. Osgood- Schlatter disease

B. Legg- Calve-Perthes disease

C. Sipped capital femoral epiphysis ( SCFE)

D. Tibial torsion

70. The recommended treatment for a compound fracture of the tibia is :

A. Internal fixation by a plate and screws.

B. Plaster fixation.

C. External skeletal fixation.

D. Intra-medullary nail fixation.

E. Skeletal traction

71. A 10 year old boy involved in an accident. X-ray shows fracture neck fibula, on
examination he is unable to dorsi- flex the ankle , the nerve involved is :

A. Popliteal nerve .

B. Anterior tibial

C. Lateral popliteal (common peroneal) nerve.


D. Superficial peroneal nerve.

E. Sciatic nerve.

72. 3rd degree Pott's differs from 2nd degree in:

A. Site of displacement of talus.

B. Fracture of medial malleolus .

c. Fracture of posterior malleolus.

D. A and C.

E. All of the above

73. A marchfracture most frequently results from:

A. Direct trauma.

B. Jumping from a height.

C. Metatarsal fracture from prolonged walking.

D. Use of high- heeled shoes.

E. Osteoporosis.

74.Fracture of calcaneus bone is usually associated with :

A. Fracture or dislocation of the hip.

B. Fracture of femur.

C. Tibial fracture.

D. Spine fracture

75. The following are predisposing factors for acute hematogenous osteomyelitis except :

A. Young age.

B. Female sex.

C. Bad hygiene.
D. Epiphyseal trauma

76. Regarding osteomyelitis all the followings are true , Except:

a.Most commonly due to staph aureus infection

B. Infection usually affect the metaphysis of long bones

C. Dead bone within the medullary canal known as involucrum

D. May need surgical drainage if there is no response to antibiotic after 48 hour

77.Surgical treatment of acute hematogenous osteomyelitis :

A. Indicated if there is no response to antibiotic for 1 week.

B. Better as it drains pus .

c. of no value.

D. Indicated in all cases.

78. Differential diagnosis of acute hematogenous osteomyelitis includes the following


except:

A. Ewing's sarcoma.

B. Osteoclastoma.

C. Septic arthritis

D. Cellulitis.

79. The following tumor must be differentiated from acute hematogenous osteomyelitis:

A. Osteoclastoma .

B. Osteosarcoma .

C. Ewing's sarcoma

D. Multiple myeloma .

80. As regards complications of acute hematogenous osteomyelitis all are correct except:
a.It can always result in suppurative arthritis

b.Chronic osteomyelitis is a common complication that can lead to handicapping.

c. Pathological fracture may occur.

D. Disturbed bone growth may lead to limping

81. The following are radiographic findings in chronic hematogenous osteomyelitis except:

A Periosteal elevation.

B. Sequestrum (hyper dense lesion)

c. Onion peel appearance .

D. Involucrum

82. The sequestrum in X-ray appears:

A. Dense .

B. Light .

c. fsodense as surrounding bone.

D. Any of the above

83. About Brodie's abscess:

A. Presented by intermittent pain after effort

B. Treated by saucerization and grafting by bone chips.

C. is a chronic abscess.

D. Apple jelly pus is often drained and is sterile

E. All of the above

84. Regarding tuberculosis of the spine all are true, EXCEPT:

A. It is secondary tuberculosis.

B. Disease starts in anterior vertebral margin.


C. Commonly affects cervical spine.

D. It may be complicated by paraplegia.

85. The earliest sign of TB hip in X-ray is:

A. Narrow joint space .

B. Irregular moth-eaten femoral head

c.Periarticular osteoporosis .

D. Dislocation

86. In Pott's spine , the disease starts in the:

A. Intervertebral disk.

B. Anterior vertebral margin.

c. Posterior vertebral margin .

D. Paravertebral soft tissue

87. The radiological signs of Pott's disease include the following except:

A. Wedging of vertebral bodies.

B. Decalcification and rarefaction of affected segment.

C. Intact intervertebral discs.

D. Angular kyphosis.

E. Soft tissue shadow due to cold- abscess formation.

88. About Pott's disease of the spine, all the following statements are true, except:

A. This is tuberculous osteomyelitis of the spine.

B. It is a disease of childhood.

C. Bones show bone destruction without new bone formation.

D. A cold abscess may show as a swelling in the thigh.

E. Treatment is mainly surgical


89. Bone cysts are common in:

A. Femur

B. Tibia.

C. Humerus.

D. Radius

90. Osteoid osteoma originates from:

A. Periosteum.

B. Cortex.

C. Medullary cavity.

D. All of the above.

91. 19-year-old man presents, having noticed a hard lump above his knee. He initially
noticed the lump 3' years ago but presents now after his girlfriend persuaded him to find out
what it is. He dose not complain of any associated symptoms. Radiographic investigation
demonstrates a knob of bone on the surface of the distal femur, which projects away from
the knee joint The most likely diagnosis is

A. Ossification of quadriceps tendon

B. Exostosis

C. Enchondroma

D. Osteoma

E. Osteosarcoma

92. The following statements about osteochondroma (exostosis) are true except:

A. Rarely affects fiat bones.

B. Can lead to mechanical block of the near joint.

C. Affects the diaphysis of long bones

D. May be associated by dwarfism .

E. Can be multiple

93. Regarding enchondroma, all of the following are true EXCEPT:


a. it is a benign osteogenic bone tumor .

B. The hand is a common location .

C. may undergo malignant transformation

D. It may cause a pathologic fracture .

E. Calcification are characteristic x ray findings

94. Concerning osteoclastoma, the following statements are correct except that it :

a. Usually occurs between the ages of 15 and 40 years.

B. Always arises in the metaphyseal region of cartilagenous bones.

C. Consists of large giant cells in a very vascular stroma of spindle cells.

D. Presents as a painless globular swelling with well-defined edge

e.Produces diagnostic radiological signs.

95. The treatment of osteoclastoma includes the following measures except:

A. Curettage of tumor tissue and packing cavity with bone chips.

B. Excision with safety margin of bone.

c.Amputation.

D. Radiotherapy

E. Chemotherapy

96. The main presentation of osteosarcoma is:

A. Pain

B. Swelling

C. Fever

D. Hemoptysis

97. All of the following are true about bone tumors except:

A. Patients with a past history of malignancy who present with backache have metastases
until proved otherwise.

B. The extent of metastases can be best assessed on a bone scan.


C. All patients with suspected bone tumors should have plain x-rays .

D. Pathological fractures through metastases should be fixed but patients should not be
given radiotherapy as this will prevent healing

98. Ewing's sarcoma is characterized by the following except that it

A. common tumor of children

B. Always arises in the metaphysis of a long bone.

C. Presents as a fusiform swelling with inflammatory changes in the overlying soft tissues .

d.May be associaed with leucocytosis .

E. Produces characteristic radiological signs.

99. A year old boy presents with loss of weight, pain and feverOn examination, a soft tissue
mass is palpable over the mid-thigh regionX-ray shows a large soft tissue mass with
concentric layers of new bone formation, the diagnosis will be:

A. Osteosarcoma

B. Osteoclastoma.

C. Ewing's sarcoma.

D. Leiomyosarcoma

100. Which of the following is the most common malignant lesion of the bone:

A. Chondrosarcoma.

B. Fibrosarcoma.

C. Ewing's sarcoma.

D. Osteosarcoma

101. Sun ray appearance of osteosarcoma is because of:

a. Periosteal reaction.

B. Osteonecrosis.

C. Calcification along vessels.


D. None of the above

102. Which of the following tumors arises from the metaphysis of long bones?

A. Osteosarcoma.

B. Giant tumor

C. Secondaries.

D. Chondrosarcoma 1

103. Regarding chondrosarcoma :

A. Most commonly affects the flat bones.

B. Is radio sensitive.

C. Has onion peel appearance as a characteristic radiological appearance .

D. Commonly spread to the regional draining lymph nodes.

E. Occurs most often in children.

104. The most common malignant bone tumors:

A. Osteosarcoma.

B. Ewing's sarcoma.

Osteoclastoma.

D. Secondaries

105. The following tumor is locally malignant and more common in females:

A Osteoclastoma .

B. Osteosarcoma .

C. Ewing's sarcoma

d.Multiple myeloma .

106. As regards skeletal metastasis from cancer prostate all are correct except:
a. May be osteogenic .

B. Can't lead to pathological fractures .

c. Is the most common primary source for bone metastasis in males.

D. Leads to marked elevation of acid phosphatase

107. About bone metastases, all the following statements true, except:

A They can cause spinal cord compression

B. They can cause hypercalcemia

c. Alkaline phosphatase is .

D. Whole body scan for deposits done by MRI.

E. Biphosphonates reduce the incidence of pathological fractures these patients .

108. The following statements about , cubitus valgus deformity are correct except that:

a.It may be due to malunited supracondylar fracture of the humerus or non united fracture
of the lateral condyle.

B. The deformity is most obvious when the elbow is fully flexed.

c. It predisposes to delayed ulnar neuritis.

d. Treatment by supracondylar osteotomy is necessary only when the deformity is severe .

109. the following regarding club foot are false EXCEPT :

A It is an adduction deformity of the forefoot.

B. is congenital deformity of the hip joint.

c.The first line of treatment is surgical correction .

D. It is always a bilateral condition

E. It corrects spontaneously by walking age.

110. Delayed ulnar neuritis is due to:

A Neurofibromatosis
B. Elbow dislocations

c.Cubitusvarus deformity

D. Cubitus valgus deformity

111. Looser's zones (also known as pseudofractures ) are a radiographic characterizing which
one of the following bone diseases?

A. Osteomalacia

B. Osteoporosis

c.gout

D. Paget's disease

E. Osteomyelitis

112. First bone to ossify foetal :

A. Femur

B. Tibia

C. Clavicle

D. Sternum .

113. The Ideal treatment of anterior elbow dislocation is:

A. Closed reduction .

B. ORIF

c. Skin traction

D. None of the above.

114. For prevention of Sudek's atrophy patients with Colle's fracture should start
physiotherapy :

A. From the first day.

B. After one week.

C. After removal of cast.


D. After 9 months

115. A fracture to the ulna with associated dislocation of the radial head is called:

A. Morgagni's fracture.

B. Galeazzi's fracture.

C. Monteggia's fracture.

D. Colles' fracture

116. The incorrect statement about anterior dislocation of the shoulder joint is:

A. The shoulder loses its round contour and become flattened.

B. The elbow is abducted from the sides.

C. All movements of the shoulder are limited and painful.

D. The anterior and posterior folds of the axilla are elevated.

E. The hand cannot be elevated on the opposite shoulder

117. Bennett's fracture is :

A. reversed Colle's' fracture

B. fracture of the scaphoid bone in the wrist

C. fracture of the radial styloid (chauffeur's fracture)

D. fracture dislocation of the first metacarpal

118. Malgaigne fracture is:

A. Avulsion fracture of ASIS.

B. Unilateral fracture of ischeal ramus.

C. Double pelvis fracture on one side with upward dislocation.

D. None of the above.


119. About fracture neck femur:

A. Garden classification depends on age of patient .

B. Intracapsular fracture has better prognosis than extracapsular .

C. Pertrochanteric fracture mean fracture of both greater and lesser trochanters.

D. Impacted fracture carries greater risk of avascular necrosis of head .

E. None of the above

120. A 30 years old woman has posterior pelvic fracture , tachycardia and hypotension ,
responding poorly volume replacement . US revealed free intraperitoneal bleeding and a
pelvic hematoma . The appropriate management is :

A. Application of medical antishock measures .

B. External fixation to stabilize the pelvis .

C. Laparotomy and pelvic packing .

D. Laparotomy and ligation of iliac arteries

121. As regards acute pyogenic arthritis all are correct except:

A. Differentiated from acute hematogenous osteomyelitis by loss of passive movement

B. Treated primarily by surgical drainage .

c. Usually complicate acute hematogenous osteomyelitis .

D. May be of iatrogenic origin.

e.Caused usually by Staph aureus transmitted through blood stream.

122. The following statements about multiple myeloma are true except that :

A. Is a primary malignant tumor of bone marrow .

b.Characterized by the presence of Bence Jones proteins in urine

C. Is rarely associated with anemia

D. May cause paraplegia .

E. Can produce abnormal immunoglobulins


123. Which cell is primarily responsible for production of alkaline phosphatase:

A. Fibroblast

b. Chondroblast.

C. Osteoclast .

D. Osteoblast

E. Hepatocyte.

124.A 7year-old child presented with intermittent limp and pain in the right hip and knee.
On examination, flexion and extension movements were free and there was no tenderness
and no muscle wasting. X-ray examination confirmed the diagnosis of

A. Early tuberculous arthritis of the hip joint

B. Traumatic arthritis

c.Perthes' disease .

D. Slipped upper femoral epiphysis.

E. Coxavara.

125. A 60year-old male with 3 months history of severe back-ache, anemia and loss of
weight developed severe girdle pains with weakness of the lower limbs. Examination
revealed low grade fever with marked tenderness over the spine, ribssternum, skull and
pelvic bones. X-ray examination of the skeleton revealed multiple punched out defects
without any new bone formation. The most probable diagnosis is:

A. Bone metastasis from an occult primary

B. Multiple myeloma.

C. Osteitisfibrosacystica

D. Hand Schuller-Christian's disease .

E. Paget's disease.

126. Paralytic talipes is differentiated from congenital talipes by the following features
except that:

A The deformity appears later after birth.

B. The limb is atrophied, cyanosed and cold.

C. The muscles are wasted and flabby


d.Usually both sides are affected.

E. The deformity can be corrected easily by manipulation .

127. of the following are true about Perthe's disease except:

A. It is a spontaneous avascular necrosis of the hip.

B. It is most common in boys around puberty.

C. The condition usually settles spontaneously .

D. A similar problem can occur in children with sickle cell disease .

E. May require surgical containment with a sub trochanteric osteotomy

128. Regarding Perthe's disease choose the correct answer:

A. Usually presents after 10 years of age

B. Is more common in girls

C. Is due to avascular necrosis of the distal femoral epiphysis

D. Plain x-ray may show the capital femoral epiphysis to be smaller, denser and flatter.

129. Regarding Colles' fracture, all are true EXCEPT:

A. The fracture may be complicated by carpal tunnel syndrome.

B. It is wise to screen for osteoporosis.

c.Non -union is a common complication .

D. Dinner fork deformity is a characteristic clinical finding

E. is caused usually by a fall on the outstretched arm.

130. In an adult patient with fracture shaft of femur :

A. Up to two liters of blood can be lost in the thigh without obvious swelling or bruising.

B. Fat embolism can occur as early as few hours after injury.

C. Distal absent pulses should be treated by immediate heparinization


D. Risk of deep venous thrombosis in the postoperative period is excluded if internal fixation
was done properly .

131. Regarding malignant bone tumours all are true, EXCEPT:

A. The most common is secondaries.

B. Secondaries are more common in hand & foot than in pelvic & shoulder girdles.

C. Bone secondaries are more commonly osteolytic than osteosclerotic in nature.

D. The peak age of occurrence of multiple myeloma is 50-60 yrs.

E. Ewing's sarcoma is both radio Schemosensittve.

132. The most common benign tumor of the rib is:

A. Osteochondroma.

B. Fibrous dysplasia.

C. Chondroma.

D. Eosinophilic granuloma

133. Regarding TB of hip joint all are true, EXCEPT:

a. Hip is second common site of affection by tuberculosis after spine.

B. Tuberculosis lesions may start in the acetabulum or metaphysis (Babcock's triangle).

C. Limping is the earliest and most common symptom of tuberculosis of the hip.

D. It heals by bony ankylosis.

134. After a breech delivery of a boy, the attending nurse noted that there is deformity of
the right thigh, upon which plain X-ray was done and fracture femur was diagnosed . Which
of the following will be used in the management :

A. Thomas' splint

B. Open reduction

C. Crede's method .

D. Gallow's Traction
135. Regarding multiple myeloma all are true, EXCEPT:

A. Is a tumor made up of malignant monoclonal plasma cells.

b.Usually affects patients > 40 years old.

c. Patients often present with malaise, bone pain, or pathologic fractures.

d.The classic radiographic appearance is multiple sclerotic lesions .

136 :Ewing's sarcoma arises from which of the following:

A. Squamous cells

b.Mesothelial cells

C. Endothelial cells

D. None of the above

137. The treatment of clubfoot should best begin at:

A. Day of birth

B. One month

C. Three month of age

D. Sixth month of age

138. Regarding Multiple myeloma all are true Except:

a.Usually affects old males

B. Related to plasma cells

C. Presented with pallor, pathological fractures and paraplegia

D. X-ray shows osteogenic areas of skulf and

139. Perthes the movements restricted are:

A. Abduction and external rotation

B. Abduction and internal rotation

C. Adduction and external rotation


D. of the above

140. True regarding Sudek's atrophy of hand:

A. Hands are painful and swollen .

B. Osteoporosis of carpals and metacarpals.

C. There is decrease in blood supply to para-articular area.

D. Cervical sympathectomy may be of help.

E. All of the above

141. Pelvic fracture are most commonly associated with visceral injury to which of the
following:

A .Prostatic urethra

B. Urinary bladder

C. Ureter

D. Uterus

142. the most common site of spine injury after trauma is:

A. Lumbosacral spine

B. Cervical spine.

C. Dorsal spine.

D. Dorsolumbar spine.

143. An elderly anemic patient complains of back pain, numerous punched out
radiolucenies on skull radiograph, the patient should be suspected of having:

A. malignant lymphoma

B. Ewing's sarcoma

c.thalathemia

D. Multiple myeloma
144. A bone malignancy characterized by appearance of Bence Jones protein in urine:

A. Osteosarcoma

b. Chondrosarcoma

C. Giant cell tumor

D. Multiple myeloma

145. Which of the following statements regarding the holding of fractures is true?

A. Treating a fracture in traction leads to delayed healing B. Plates and screws do not allow
absolute stability to be obtained.

C. Intramedullary nails should always be used in growing bones.

D. External fixators are especially useful where there is loss of soft tissue

146. An elderly woman falls and sustains an oblique fracture of the mid- shaft of the
humerus. Choose the optimum method of stabilization ?

Collar-and-cuff traction

147. A young motorcyclist sustains a fracture of the tibia where the skin and muscle have
been ripped off over a distance of 10 cm. choose the optimum method of stabilization?

External fixation

148. An elderly patient falls and sustains a displaced sub- capital fractured neck of femur.
Choose the optimum method of stabilization?

Hemiarthroplasty

149. An elderly patient falls and sustains a displaced per- trochanteric fractured neck of
femur. Choose the optimum method of stabilization?

Dynamic hip screw


150. A young man falls and sustains three broken ribs. choose the optimum method of
stabilization:

Answer: analgesia

151. A patient develops severe and sudden onset of pain in the shoulder after a minor
trauma. movements are restricted, She cannot sleep for the pain. Choose correct treatment

Answer: analgesia and gentle physiotherapy

152. A middle- aged woman gets sudden onset of pain in her shoulder after no trauma . She
is tender anterolateraly the pain is severe that she cannot sleep. She is able to rotate the
shoulder externally without much pain, but other movements are painful , especially active
ones. -ray shows calcification floating between the acromion and the humeral head Choose
correct treatment:

Answer :steroid injection

153. A patient attends with pain over the lateral side of the elbow following a weekend
redecorating the house . Choose correct diagnosis:

Answer: tennis elbow

154. A patient with rheumatoid arthritis attends with pain in the elbow mainly on pronation
and supination. Choose correct diagnosis:

Answer : Arthritis of the radio-humeral joint

155. A patient with rheumatoid arthritis attends with pain and weakness in the elbow and
significant bone destruction. Choose correct diagnosis:

Answer: Arthritis of the elbow


156. A patient who broke their elbow many years ago now presents with pain and
numbness down the medial side of the forearm into the little finger. Choose correct
diagnosis:

Answer: Ulnar nerve entrapment

157. A patient presents with a red hot lump over the back of the elbow. Choose correct
diagnosis :

Answer: Olecranon bursitis

158. A patient has a chronic hot swollen and painful and a low- grade fever. Choose correct
diagnosis:

Answer: Tuberculosis

159. Which of the following bone tumors is characterized by a11:22 translocation?

a. Giant cell tumor of the bone

B. Chordoma

C. Chondroblastoma

D. Ewing's sarcoma

160. Lateral stability of the ankle is provided by :

A. The tarsal navicular

B. The medial meniscus


C. The deltoid ligament

d.The fibula

161. The most appropriate ttt for posterior dislocation of the sterno - clavicular joint is:

A. Analgesics only

B. Ipsilateral sling to immobilize the shoulder

C. Closed reduction

D. Open reduction and internal fixation

162.Which of the following is most diagnostic for a compartment syndrome ?

A. Compartment pressure >25mmHs

B. Compartment pressure 20 mmHg, higher than diastolic pressure

c. Tense extremity on palpation

D. Disproportionate pain and painful passive stretch of the compartment muscles

163. Which of the following is the preferred treatment for a Monteggia fracture dislocation
of the proximal forearm?

A. Closed reduction of both ulna and radius

B. Closed reduction of , open reduction and fixation of radius

c.Open reduction and fixation of ulna, closed reduction of radius

D. Open reduction and fixation of the ulna and radius

164. A 4-year -old presents with a fracture of mid shaft femur . Which of the following is the
best treatment for this child?

A. Traction and bed rest

b. Closed reduction and application of a spica cast

C. Open reduction and internal fixation (plate)


D. Open reduction and intramedullary fixation

165.Which of the following is the most common cervical fracture seen after a driving
accident?

A. Dens (odontoid ) fracture (C1)

B. Hangman fracture (C2)

C. Compression fracture (C3-7)

D. Burst fracture (C3-6)

166. For the patient described in the previous question, urine did not extend to the leg
because the membranous layer (Scarpa's fascia) is fused inferiorly with which of the
following?

A. Femoral sheath .

B. Fascia lata

C. Femoral fascia

d. Deep inguinal ring

E .Superficial inguinal ring

167. A 60year-old man is a frontseat passenger a car crash. He is found to have three
fractured ribs on the right, rupture of the liver, pelvic fracture, right femoral fracture, and a
left tibial fracture. The patient is given broad-spectrum antibiotics, and his injuries are
managed by surgery, requiring 12 U of blood. The patient improves initially, but on the third
postoperative day, he develops hypoxia (PaO2 , mm Hg), with confusion, tachypnea, and
petechia. What is the most likely diagnosis ?

A. Recurrent intra-abdominal hemorrhage from dilutionalthrombocytopenia

b. Transfusion reaction

c. Antibiotic allergy

d. Fat embolism

E. Disseminated intravascular coagulopathy (DIC)


168. A newborn boy was examined to exclude congenital dislocation of the hip (CDH)Which
of the following tests is relative to the management of CDH?

A. The diagnosis should be established between 2 and 4 years of age.

B. Abduction of the flexed hip causes a click (Ortolani's sign)

C. Abduction of the hip is not limited

D. Apparent lengthening of the thigh with the hip and knee fiexed may be seen.

E. Open reduction usually is required.

169. A 50-year-old man hears a "snap" and then feels pain in his right leg while lunging
forearm drive playing tennis. He walks off the court with difficulty, but leg is swollen and
painful. Findings on - rays of the leg and ankle in the emergency room negative. Foot
sensation is normal, findings on the Thompson test of plantar flexion to occur after
squeezing the gastrocnemius) are positive. What is the diagnosis?

A. Gastrocnemius muscle tear

b. Acute thrombophlebitis

c. Rupture of the Achilles tendon

D. Acute compartment syndrome

E. Fibula fracture

170. A 40-year-old housewife trips over the garden hose, landing on the patio with an
outstretched hand. Swelling and pain in the wrist rapidly occur, but findings on emergency
room x- rays are negative for fracture or dislocation. In addition to the swellingthere is
restriction of wrist and flexion as well as some tendemoss of the anatomic snuffbox at the
base of the thumbWhat is the best treatment?

a.Splint the wrist for 4 days until the swelling and wrist pain subside.

B. Apply a cast to the wrist and repeat the wrist x-ray in 10-14 days .

C. Apply a cast to the wrist for weeks .

D. Apply an Ace wrap to the wrist and remove range of motion and exercise in warm water.
E. Perform open exploration of the wrist

171. A 55-year-old right- handed woman has left elbow pain laterally after cleaning up a
flooded basement by wringing out water-soaked rags. X-ray findings are negative. There
tenderness and slight swelling over the lateral epicondyle of the humerusAnatomically this
condition can be explained by which of the foffowing?

A. Sprain of the lateral collateral elbow ligament

B. Rupture of the triceps muscle

c. Tendinitis of the wrist extensors

d. Synovitis of the left elbow joint

E. Rupture of pronator teres muscle

172. A 16-year-old cross-country runner experiences right mid-leg pain during workouts,
Sometimes the pain prevents him from completing the prescribed mileage. There is mid
tibial tenderness but no deformity. ROM of the ankle and knee are full and painless. There is
no calf tenderness or fullness, and the Achilles tendon is intact. X-ray findings for the tibia
and fibula, including both the ankle and knee joints, are normal. What should the patient be
advised to do?

A. Rest, take anti-inflammatory agents, and use crutches for 2 weeks.

B. Wear a short leg cast for 3 weeks

C. Resttake anti-inflammatory agents, use crutches, and undergo a bone scan.

D. Continue running but increase stretching exercises before and after workout and apply
analgesics to the painful area for 20 minutes after workout.

E. Use steroids .

173. An 86-yearold woman experiences left hip pain after a fall at home . She cannot
ambulate , her hip area is swollen and painful , and her left lower extremity shortened and
externally rotated . Before the fall, she was ambulatory and had no complaint of hip, pelvic ,
pain . In addition to the fracture of the proximal portion of the left femur , the -ray would
show which of the following ?

A. Arthritis of the left híp

B. Calcific bursitis of the left hip

c. Osteoporosis
D. Fracture of the pelvis

E. Dislocation of the head of the femur

174.assessment of a patient in the outpatie department you identify that they are
Trendelenburg's test positive. This indicates a possible palsy of which nerve?

a. Sciatic

B. Femoral

C. Obturator

D. Superior gluteal

e. Inferior gluteal

175. The most common cause of an acquired valgus deformity of the knee joint is:

A. Rheumatoid arthritis

B. Osteoporosis

C. Trauma

D. Osteoarthritis

E. Osteomalacia

176. Dupuytren's contracture is caused by a thickning of the palmar fascia is associated with
the following conditions except:

a. Alcoholic cirrhosis

B. Peyronie's disease

C. Acquired immune deficiency syndrome

D. Epilepsy

E. Syphilis

177. Your registrar tells you that his patient has a boxer's fracture. From this, you know that
the bone which is fractured is:

a. The fifth metacarpal


B. The fourth metacarpal

C. The fifth proximal phalanx

D. Hamate

E. Styloid process

178. A patient presents with symptoms suggestive of adhesive capsulitis ( frozen shoulder )
Which of the following systemic conditions is most commonly associated with this?

A. Fibrotic lung disease

B. Systernic lupus erythematosus

C. Sjogren's syndrome

D. Osteoarthritis

E. Diabetes

179. A 37-year- old patient presents to you having noticed a lump on the lateral aspect of his
feg. The lump was first noticed while the patient was in the gym on a rowing machine. There
is associated painOn examination when the patient tenses his relaxed. With the leg relaxed,
it is possible to identify a depression in the fascia lata. The diagnosis :

A. Intramuscular haematoma

b.Partial quadriceps rupture

c. Muscle hernia

D. Intermuscularlipomata

E. Myosarcoma

180. Which one of the following statements regarding Paget's disease of bone is not correct
?

A. it most commonly occurs in the pelvis

B. It more commonly affects a single bone

c. it occurs in 3 per cent of all individuals over 40 in the UK

D. Characteriste findings include raised serum alkaline phosphatase, normal calcium and
normal phosphate
E. 1 per cent of cases will develop osteosarcoma

181. -year-old Alrican Caribbean boy is referred to the orthopedic team with an acutely
painful arm. On examination there is a notable swelling of the limb above and around the
elbow joint. There is no reduced range passive movement of the joint but the boy is holding
his arm very still and will not actively move the limb. Investigations include haemoglobin B.3
white coll count 10.5 109/, C- reactive protein 12mg / L The child is eats / m * i * n apyrexial
with a blood pressure of 110/75 mmHg, and a pulse rate of 85 beats/min

A .Septic arthritis

B .Osteomyelitis

C .Bony infarct

d. gout

E. Juvenile arthritis

182.The least possible complication of an unstable fracture of the pelvis is?

A .Severe bleeding

B .Urinary bladder injury

C .Injury of the urethra

d.Malunion that causes future obstruction of the birth canal

E. Femoral nerve injury

183. Regarding multiple myeloma, pick up the wrong statement:

A. Blood picture may show plasma cell leukemia

B .They arise from the plasma cells.

C .The distal end of the femur and the proximal end of the tibia is a common site .

D. Bence Jones protein coagulates at 55 degrees * C and disappears at 85 degrees * C


184. Regarding the site of bone tumors, pick up thewrong statement :

A .Metaphysis , secondaries

B. Chondrosarcoma , scapula

C. Osteoblastoma ,vertebrae

d. Diaphysis, Ewing's sarcoma

185 Osteoclastoma is characterized by all except:

A .The usual presentation is 20-30years

b. The lesson is metaphyseal

C. The usual presentation is a bony mass

d. The commonest site of the lesion is around the knee

E. reveals a soap bubble appearance

186. The recommended treatment for a closed fracture mid shaft femur in an adult is:

A. Reduction and fixation in a plaster cast.

B. External skeletal fixator.

c. Intra-medullary nail.

D. Skeletal traction.

E. Thomas splint.

187. All of the following are correct about giant cell tumor of the bone except:

A. it causes expansion of the affected area

B. It usually arises in the diaphysis of the affected bone

C. Surgery and radiotherapy have got a role in the treatment.

D.The most commonly affected bones are around the knee.

E. Characterized by the radiological appearance of medullary plug.


188. The local complications of closed fractures does not include:

A. Malunion .

B. Non union

C. Osteomyleitis

d.Sudek's atrophy

E. Joint stiffness .

189.Which of the following lesions does not commonly cause metastasis in the bones:

A. Renal cell carcinoma.

B. Prostatic cancer .

C. Follicular carcinoma of the thyroid.

D. Rectal carcinoma .

E. Breast cancer.

190. Which statement is untrue concerning Sudek'satrophy :

A. Occurs most often after wrist and ankle injuries.

B. Is characterized by severe pain and stiffness.

C. Followed by osteoarthritis of the near joint

d.. May require sympathectomy.

E. a type of osteodystrophy.

191, Regarding TB of the dorsal spine :

A. Usually due to direct spread from a near by focus.

B. Lumbar lordosis is a common feature .

C. The inter vertebral discs commonly spared.

D. Can leads to the formation of a psoas abscess.

E. Can be treated with skeletal traction .


192. Which of the following fractures are more common to be impacted:

A. Neck femur fracture.

B. Humeral supracondylar fracture.

C. Colle's fracture .

d. Clavicle fracture.

193. Which of the following are factors affecting bone healing:

A. Type of bone

B. Intraarticular fracture

c. drugs

D. Surgical intervention

E. All of the above

194. Mafunion may result in the following except:

A. Angulation.

B. Rotation

c.Elongation.

D. Shortening

195. indicated in the following except:

A. Unstable fractures

B. Open fracture

C. With neurovascular injuries (when exploration is needed)

D. Pathological fractures.

196. The followings are complications of fracture except:


A. Neurogenic shock.

B. Stroke.

C. Fat embolism

d. Renal failure.

E. Sepsis .

197. The following types of shock may complicate femoral fracture except:

a. Hypovolemic.

B. Neurogenic

C. Cardiogenic

D. Septic

198. Immobilization of fractures of long bones should include :

a. Fractured bone only

b. Joint Involved in the fracture

C. Proximal joint

d. Both proximal and distal joints

E. Distal joint

199. The most common type of shoulder dislocation is:

A. Posterior

B. Anterior

c.Inferior

d. Superior

200. The commonest complication of anterior shoulder dislocation is:

A. Axillary artery injury .


B. Rotator cuff tear.

C. Recurrent dislocation

d. None of the above.

201. Anterior elbow dislocation is usually associated with fracture of :

a.Coronoid process

b. Olecranon process

C. Head of radius

D. None of the above

202. All of the following are common complications of anterior shoulder distocation except:

A. Rotator cuff tear.

B. Radial nerve injury

C. Axillary nerve injury.

D. Brachial plexus injury.

E. Recurrent dislocation

203. Most important complication in fracture both bones of forearm :

a.Median nerve injury.

B. Madlung deformity.

C. Compartmental syndrome.

D. Carpal tunnel syndrome.

204. Most common fracture of upper limb is:

a.Supracondylar fracture.

B. Shaft humerus fracture.

C. Clavicle fracture.
D. Colle's fracture

205. Most common joint to be dislocated is:

A. Hip

B. Knee.

c. Elbow.

D. Shoulder .

206. Recurrent dislocation is common complication in :

A. Hip dislocation .

B. Knee dislocation

C. Anterior shoulder dislocation .

D. Posterior shoulder dislocation .

E. Elbow dislocation

207. The following are deformities that can result from Colle's fracture except :

A. Madlung deformity .

B. Dinner fork deformity .

C. Ape hand deformity .

D. Wrist drop

208. The most common hip dislocation is:

a.Congenital dislocation.

B. Traumatic dislocation.

c. Inflammatory dislocation.

D. Paralytic dislocation.
209. Lengthening is a possible sign of:

A .Fracture neck femur.

B. Fracture hip bone.

c. Anterior hip dislocation.

D. Posterior hip dislocation.

210. The most important predisposing factor in fracture neck femur is:

a. Smoking.

B. DM.

C. Postmenopausal osteoporosis.

D. Poliomyelitis.

211. The 1st step in management after clinical diagnosis of Pott's fracture is:

A. Searching for associated injuries.

B. Trial to reduce dislocated ankle.

C. X- ray for accurate diagnosis and exclusion of associated injuries.

D. Proceed to fixation according to clinical diagnosis.

212. As regards Irreducibility in posterior hip dislocation all are correct except :

A. May lead to handicapping in neglected cases.

B. May be due to button hole tear of joint capsule

C. May be due to entrapment of fragment from associated fracture acetabulum

D. May be due to spasm of muscles around the joint

213. The following factor(s) contribute to morbidity and mortality in fracture neck femur:

A. Old age .

B. Prolonged recumbency
C. Osteoporosis.

D. Avascular necrosis of head.

E. All of the above

214. The most common causative organism in acute hematogenous osteomyelitis:

A. Staph.

B. Strept.

C. E.coli.

D. Salmonella typhi

215. Tuberculosis of hip joint is characterized by the following except:

A. Always have blood borne origin.

B. Presented by characteristic night pains (night cries).

c. It can lead to new bone formation (bony ankylosis).

D. None of the above.

216. Tuberculosis of the spine most likely originates from:

a. Intervertebral disk

B. Cancellous vertebral body

c.Ligamentous structures

D. Paravertebral soft tissue.

217. The following tumor is more common in females :

a. Osteoclastoma

B. Osteosarcoma

C. Ewing's sarcoma

D. Multiple myeloma.
218. In pott's disease of the spinothe following statements are correct except that it :

A. Is commonest in male children.

B. Affects the dorsolumbar region most often

C. May affect one vertebra only

D. due to blood spread of tubercle bacili from a primary focus

E. May remain silent until deformity, cold abscess or paraplegia.

219. The signs of fractured shaft of a bone do not include:

A. Swelling

B. Deformity

C. Loss of all movements in the limb.

D. Acute localized bone tenderness.

E. Abnormal mobility in the line of the bone.

220. Which statement is untrue concerning Sudek's atrophy?

A. Occurs most often after wrist and ankle injuries

B. Is characterized by severe pain and stiffness.

C. Is never associated with local vasomotor symptoms.

D. Produces characteristic radiological signs

E. May require sympathetic block or sympathectomy.

221. An eldery female sustained Colles' fracture which was properly treated. However, she
developed severe pain and stiffness of the wrist with coldness and cyanosis of the hand. X
ray examination revealed diffuse decalcification of the bones. She proved to be suffering
from:

A. Causalgia.

B. Tuberculous arthritis of wrist joint

C. Traumatic tenosynovitis
D. Sudek's atrophy.

E. Osteoarthritis of wrist joint

222. A -year-old boy presented with limping and pain in the right knee two days after a fall
in the street. On examination he looked Ill and in severe pain with high fever and swelling of
the knee region extending to the thigh which was warm and very tender. The most probable
diagnosis is:

A. Traumatic synovitis.

B. Hemarthrosis

c.Acute osteomyelitis of the femur.

D. Septic arthritis of knee

E. Bone sarcoma.

223. The most common osteolytic metastasis in bones are derived from the:

A. Lung

B. Breast

C. Stomach

D. Kidney .

E. Prostate .

224. All of the following are true about soft tissues haematomas except:

a.Most resolve spontaneously .

B. If a cyst develops surgical excision may be required .

C. The changes can become malignant .

D. The bruised muscles may sometimes be replaced by cartilage.

225. What are the characteristics of bursae ?


A. They are normal structures designed to reduce friction

B. They are lined with synovial membrane and connected to the joint beneath

C. They are able to become inflamed and infected.

D. They have no nerve supply

E. A &C

226. All of the following are true about bone tumors except:

A. Patients with a past history of malignancy who present with backache have metastases
until proved otherwise .

B. The extent of metastases can be best assessed on bone scan

C. All patients with suspected bone tumors should have plain -rays.

D. Pathological fractures through metastases should be fixed but patients should not be
given radiotherapy as this will prevent healing

227. Vollkman's ischemia commonly occurs following:

A. Fracture shaft humerous

b.Supra- condylar fracture

C. Colle's fracture

D. Fracture lateral condyle humerous 2

28. Arthroplasty defined as:

A. Replacement of the articular surface of a long bone by synthetic material

B. Correction of angulation of long bones

C. Open reduction and internal fixation of intra- articular fracture

D. Open reduction and intemal fixation of long bone fracture

E. Open drainage of an infected joint

229. Hematogenous osteomyelitis most frequently affects:


a. The diaphysis of long bones

B. The epiphysis of long bones

c. The metaphysis of long bones

D. The flat bones

230. Which of the following statements about tennis elbow is incorrect:

a.Occurs only in tennis players .

B. a strain of the common extensor origin from the external epicondyle of the humerus.

C. Causes pain on the outer side of the elbow aggravated by dorsiflexion of the wrist.

D. May recover spontaneously within one year.

E. Is best treated local injection of cortisone acetate.

231. Regarding acute osteomyelitis all are true EXCEPT:

A .History of trauma to the affected limb is common.

B. Pathology includes sequestrum formation.

C. Pathology includes a subperiosteal abscess.

D. Pathology starts in the metaphysis.

E. X-rays on the first day show no positive findings

232. In chronic osteomyelitis, " involucrum" refers to:

A. New bone formed to replace dead bone.

B. Separated necrotic (dead) bone.

C. A cavily filled with pus and necrotic tissue.

D. Sinus discharging pus.

E. Malignant transformation .

233. Which of the following tumors arises from the metaphysis of ?


A. Osteosarcoma

B. Giant cell tumor

C. Secondaries.

D. Chondrosarcoma.

234. Which of the following statements about open fractures is False:

A. Intravenous antibiotics should be administered as soon as possible.

B. They should be regarded as an emergency.

c.Wound closure is necessary within 8 hours.

D. Systematic wound debridement and irrigation should be performed .

235. The goals of proper bone fracture reduction include the followings except:

A. Providing patient comfort and analgesia.

B. Allowing for restoration of length of the extremity .

C. Correcting angular deformity rotation

D. Allowing for some overlapping of the distal over the proximal segment of the fractured
long bone.

236. Non-union of fractures is caused by all the followings , EXCEPT:

a.Separation of the fracture fragments

B. Interposition of tissue

C. Angulation at the site of fracture

D. Poor blood supply

237. The neurovascular structure most commonly injured as a result of an anterior


dislocation of the shoulder is the:

A. Musculocutaneous nerve.

B. Axillary nerve .
C. Axillary artery .

D. Median nerve

238. The radial nerve is at greatest risk for injury with which fracture?

A. Fracture of the surgical neck of the humerus .

B. Fracture of the shaft of the humerus .

C. Supracondylar fracture of the humerus .

D. Olecranon fractures .

239. Regarding fracture surgical neck of humerus all are true, Except :

A. Occurs due to fall on outstretched hand .

B. Common in children .

C. Osteoporosis is an important risk factor

D. May be complicated by injury of axillary nerve.

240. All the following may complicate supracondylar fracture humerus , EXCEPT :

A. Median nerve injury .

B. Volkmann's ischemic contracture .

C. Cubitusvarus

D. Myossitisossificans

E. Brachial plexus palsy .

241 In fractures of the femoral neck , only one of the following is false:

A. Commonly occur in old age due to associated osteoporosis .

B. The patient cannot lift the extended lower limb .

C. Associated with external rotation & shortening of the limb.

D. Conservative treatment hip spica ) is the treatment of choice .


E. May be complicated by avascular necrosis of the femoral head .

242. Fracture neck femur in the elderly :

A. Usually results from direct trauma to the upper thigh

B. External fixation is usually adequate in most patients

C. Surgical replacement of the femoral head with prosthesis should be done as soon as
possible

D. Traction in bed for 3 months is the ideal treatment for patients over 70 years .

243. A 12-year -old boy presents with pain in his lower limb . Plain X-ray shows a sunburst
appearance with bone destruction , soft tissue mass and new bone formation of the
metaphysic of the femur . The most probable diagnosis is:

A. Osteoma

B. Ewing sarcoma

C. osteosarcoma

D. Ostoeidosteoma

244. Choose the incorrect statement about osteomyelitis :

A. The metaphysis is usually where the infection starts.

B. In children, the tibia and fibula are more commonly affected.

C. Streptococcus pyogenes is responsible for 90% of the cases.

D. In sickle cell disease, Salmonella thyphii is the most common cause.

245. As regards fracture of the clavicle, all are true, Except:

A. Usually due to a fall on an outstretched hand. 1/3 2/3 and lateral

B. Fracture usually occurs at the junction of medial of the clavicle.

C. Malunion is common.

D. Outer fragment displaced downwards and proximal fragment displaced upwards.


246. Isolated fractures of the tibia that are not displaced are usually treated by:

A. Rest.

B. Above knee plaster cast.

C. Internal fixation with intramedullary nail

D. External fixation.

247. Which of the following signs/ symptoms are pathognomonic of hyperparathyroidism ?

A. Osteitisfibrosacystica .

B. Pathologic fractures of the metacarpals .

C. Calcium oxalate nephrolithiasis .

D. Hypercalcemia causing mental status changes .

248.Nerve injury is famous for this kind of fracture :

A. Neck humerus .

B. Supracondylar humerus .

C. Colles ' fracture .

D. Neck femur .

249. Bony metastasis is famous with this cancer :

A. Renal cell carcinoma .

B. Wilms ' tumor .

C. Cancer bladder

D . Cancer prostate .

250. Crepitus is heard /felt in which of the following :

A. Osteoarthritis .
B. Fracture .

C. Subcutaneous .

D. All of the above .

251. Regarding acute hematogenous osteomyelitis , all of the following are true , Except :

A. Is caused by staphylococcus aureus in 100 % of the cases .

B. Horizontal spread of infection results in subperiostealsequestra .

C. Has to be differentiated from Ewing's sarcoma .

D. The plain -ray is normal initially .

252. Genu valgum deformity means :

A. Bow legs .

B. Knock knees .

C. Increased femoral neck shift angle .

D. Decreased femoral neck shaft angle .

E Internal rotation of the tibia .

253. Cubitus valgus means :

A. Bow legs

b.knock knee

c.inward deviation of the long axis of forearm .

D. Outward dekation of the long of the forearm.

E. Pronation of the forearm.

254. Regarding osteosarcoma , all of the following are true EXCEPT:

A. It is usually metaphyseal in location.

B. Sunray appearance is a characteristic x ray finding.


c. Clinical radiolographically it may mirnic acute osteomylitis.

D. It is a primary malignant bone tumor.

E. MRI an important diagnostic imaging tool

255. A fall on the outstretched hand can cause the following injuries EXCEPT:

a. Colles's fracture

b. Supracondylar fracture of the humerus.

c. Fracture of the scapular blade

d.posterior dislocation of the elbow

E. Fracture of the davicle

256. Regarding acute osteomylitis all are true Except:

A. commonly caused by staph aureus

b. Usually begins at the diaphysis

C. Infection is usually blood borne

D. Bone drilling is advised if rapid response to antibiotics does not occur

257. Regarding osteogenic sarcoma all are true Except:

a. Usually affects teen-agers

B. Usually affects epiphysis

C. May complicate Paget's disease

D. Usually metastasize by blood stream

258.Regarding giant cell tumor all are true Except:

a. Occurs before 20 year

b. Best treated by local excision

C. Soap bubble appearance in X-ray


D. Affects the ends of long bones

E. Arise from epiphysis

F. Pathological fracture is common

259. Regarding fracture neck femur all are true Except:

A. Commonly occurs in elderly patients with senile osteoporosis

B. Intra-capsular fractures may be complicated by avascular necrosis of the head of femur

C. Extra- capsular fractures may be complicated by mal-union

D. Elderly patient with avascular necrosis is treated lag screw

260 . Regarding fracture of the clavicle are true Except:

a.Caused by fall on outstretched hand

B. Fracture usually affects the medical third

C. The medial fragment is pulled up by steromastoid muscle

D. Malunion is of functional significance

261. False regarding intertrochanteric fracture of femur:

A. Limb shortening.

B. Malunion.

C. Avascular necrosis of femoral head .

D. Internal fixation is preferred.

262. Regarding upper limb peripheral nerve injuries, false statement is:

A. Injury to the median nerve results in wrist drop.

b. Injury to the median nerve results in loss of sensation over the palmar aspect of index
finger.

C. Injury to the radial nerve results in loss of sensation in the anatomical snuffbox.

D. Injury to the ulnar nerve results in partial claw hand.


263. A fracture is said to be:

A. Closed if an overlying skin laceration has been sutured

. B. Comminuted if there has been adjacent soft tissue damage like nerves or vessels.

c. Stress fracture, if it occurs in diseased bone.

D. Greenstick fracture in children under age of ten.

264. The commonest complication of fracture clavicle is:

A. Stiffness shoulder

B. malunion

C. nonunion

D. brachial plexus injury

265. Regarding osteosarcoma , all are true EXCEPT :

A. Is mostly a disease of adolescence and early adult life

B. Affects the epiphysis of long bones

C. Are most commonly seen around the knee and in the proximal humerus

D. Heamatogenous spread can result in pulmonary metastasis

E. Characterized by bone elevation with new bone formation

266. Which of the following statements regarding tuberculosis of the spine is false?

a. It's primary tuberculosis.

B. Disease starts in the anterior vertebral margin.

C. Back pain is the most common presenting symptom.

D. It may be complicated by paraplegia .


267. An onion peel appearance of a rib on computed tomography suggestive of:

A. Chondroma

B. Osteosarcoma

C. Plasmacytoma.

D. Ewing's sarcoma

268. A 16 boy presents with loss of weight, pain and fever. On examination , a soft tissue
mass is palpable over the mid- thigh region , X-ray shows a large soft tissue mass with
concentric layers of new bone formation , the diagnosis will be:

A. Osteosarcoma.

B.Osteoclastoma.

C.Ewing's sarcoma

D. Leiomyosarcoma

269. Which of the following tumors arise from epiphysis :

A. Ewing's sarcoma

B. Osteoclastoma (GCT)

C. Chondromyxoid fibroma

D. Osteosarcoma

270. Regarding a greenstick fracture all are true EXCEPT:

A. Seen in children

B. Breaking of cortex on one side (convex side) and bending cortex on the other side
(concave side)

C. because the bones more tough

D. Named greenstick because branches of young plant (which still fresh and) behave in a
similar fashion

271. Anterior dislocation of shoulder may be complicated by:


A. Brachial plexus injury

B. Tear of rotator cuff

C.Fracture head of humerus

D. All of the above

272. A 20 year old female presented to the clinic with a swelling around her knee joint , she
reported that she is having mild pain in her knee over the last few weeks before the swelling
appeared on examination she was feverish (38) with a firm to hard swelling in the lower end
femur -ray was done showing sun ray appearance . The possible diagnosis will be:

A. Ewing's sarcoma

B.Osteoclastoma

C.Osteosarcoma

D. Giant cell tumor bone

273. Regarding osteosarcoma :

A. Affects the epiphyses of long bones .

B. Are most commonly seen around the knee

C. Characterized by bone destruction with new bone formation

D. Most patients are above 40 years of age.

274. Which of the following statements regarding compartment syndrome is FALSE ?

A. It is detected by a loss of distal pulses and sensation.

B. It produces pain out of all proportion to the injury.

C. It is more common in closed fractures than open ones .

D. It is a result of raised pressure in a compartment collapsing the veins.

E. It is treated by a fasciotomy
275. A patient who had a breast cancer excised 5 years previously presents with weight loss
and a pathological fracture of the femur . You want to know if there are any other
secondaries . Choose the optimal imaging modality ?

Answer: Bone scan

276. A road traffic accident victim presents with a multi -fragment and displaced fracture of
the pelvis , which now needs reconstruction . Choose the optimal imaging modality:

Answer : CT scan

277. A rugby player is tackled and feels a crack in his knee. Since then, every time he turns
with his weight on that leg it is liable to give way under him, and also locks on occasions .
Choose the optimal imaging modality:

Answer: Magnetic resonance imaging (MRI )

278. A patient breaks his femur . It is treated in traction and then with a plaster brace ,
because he refuses surgery . Over a period of 8 weeks the femur bends into 15 degrees of
varus , producing a prominent lump on the outside of the thigh . Four months after the
fracture it is painless , and he can walk on it. Choose the correct diagnosis

Answer : Malunion

279. An elderly patient falls and breaks the lower end of their femur . It is fixed with a blade
plate but at review 3 months after the accident , the fracture is still visible on X- ray. Some
weeks after that the plate breaks and the fracture falls apart . A new plate is put in but this
plate also breaks some months later . Choose the correct diagnosis

Answer: Non-union

280. A young man has a fracture of the tibia in a football accident. It is fixed with a locking
intramedullary nail. At review 3 months later, the fracture site is red and hot. There is a lot
of pain and no sign of union of the fracture. Choose the correct diagnosis:

Answer :infected non-union


281. A child suffers a supracondylar fracture of the humerus after falling out of a tree. It is
treated in a plasterAt 6 weeks, the plaster is removed but it is clear that the fracture is still
mobile, although some callus is visible on X-ray . Choose the correc diagnosis :

Answer: delayed union

282. All of the following muscles control the rotator cuff except:

A. Supraspinatus

B. Teres major

C. Teres minor

D. Infraspinatus

E. Subscapularis

283. The most common bone malignancy in children is:

A. Perosteal sarcoma

B.Ewing's sarcoma

C. Osteosarcoma

D. Rhabdomyosarcoma

284. Which of the following is the radiographic findings of osteomyelitis?

A. Localized osteoporosis

B. Periosteal thickening

C.Patchy sclerosis

D. Lytic area

285. Which of the following femur fracture with significant blood loss?

A. Femoral neck fracture

B. Intertrochanteric fracture

C. Distal femoral etaphysic fracture


D. Femoral shaft fracture

286. The most appropriate initial treatment for a symptomatic osteoid osteoma of the distal
tibia is:

A. Oral anti-inflammatory medication (aspirin or NSAID)

B. Radiofrequency ablation

C.Local resection

D. Amputation

287. A 40-year-old construction worker is pulled from the rubble after a building collapses
and pins his right lower leg. X-rays in the emergency department reveal a comminuted
fracture of the right tibia and fibula. The dorsalis pedis and posterior tibial pulses are
palpable. The patient complains of severe pain that is accentuated with dorsiflexion of the
foot. The calf feels tense. What is the appropriate step?

A. ORIF of fracture

B. ORIF of fracture plus three-compartment fasciotomy

C. Closed reduction and observation

D. ORIF only if pulses become weak

E. Arteriogram

288. A football player extends his right arm to make a tackle but experiences intense pain on
tackle contact with subsequent inability to move the right arm. Examination reveals swelling
and tenderness about the shoulder with loss of the normal deltoid contour. Which is the
most likely diagnosis?

A. Brachial plexus injury

B. Anterior dislocation of the shoulder

C. Fracture of the proximal posterior portion of the humerus

D. Deltoid muscle rupture

E. Posterior dislocation of the shoulder


289. Which of the following options is not a clinical feature common to osteoarthritis and
rheumatoid arthritis of the knee?

A. Muscle wasting

B. Joint effusion

C. Baker 's cyst

D. Raised C reactive protien

E. Subcutanous nodule.

290. Rhumatoid arthritis is multisystem connective tissue disease which of the following is
not a pulmonary complication of rheumatoid arthritis ?

A. Effusion

B. Obstructive lung disease

C. Restrictive lung disease

D.Cavitation

E. Emphysema

291. All of the following are dermatological manifestation of rheumatoid arthritis except:

A. Palmar erythema

B. Erythema nodosum

C. Pyodermagangrenosum

D.Livedoreticularis

E. skin nodules

292. All of the following are causes of carpal tunnel syndrome, except:

A. Hypothyroidism

B. Amyloidosis

C. Diabetes

D. Gout
E. Rheumatoid arthritis

293. Which of the following pathological changes is not a feature of rheumatoid disease of
the hand ?

A. Boutonniere's deformity

B. Z line thumb

C. Squaring of the thumb

D. Ulnar deviation

E. Swan-neck deformity

294. A patient presents to his GP surgery complaining of a swelling on his wrist. On


examination, there is a focal swelling on the dorsal aspect of the wrist. It is smooth and non-
tender. The overlying skin is normal and moves freely over the mass, however, it seems to
be fixed to the tendon. What is the likely diagnosis:

A. Sebaceous cys

B. Lipoma

C. Ganglion

D. Giant cell tumour of the tendon sheath

E. Fibroma

295. A patient presents to the outpatients department following referral for carpal tunnel
syndrome . While taking the history and examining the patient you attempt to evaluate
whether any permanent nerve injury has occurred. Which sign is often the first indicator of
lasting nerve injury ?

A. Pins and needles

B. Thenar muscle wasting

C. Night pain

D. Reduced two- point discrimination

E. Positive Phalen's test


296. A patient with longstanding osteoarthritis presents with pain on movement of his
shoulder . You perform a full shoulder exam . You note that you can re-create the pain by
asking the patient to abduct his shoulder against resistance . You conclude that there is
impingement of which one of the following structures :

A. Teres minor

B. Supraspinatus

C. Infraspinatus

D. Subscapularis

E. Subacromial bursa

297. Out of the following bone pathologies , which is not correctly matched to the most
common site at which it occurs ?

A.Enchondroma - bones of hands and feet

B. Osteoma - cranial vault and skull

C. Ewing's sarcoma - mid -shaft femur

D. Osteosarcoma - femur , just above the knee

E.Osteomyelitis - mid-shaft long bone

298. Which one of the following organisms is most commonly associated with septic
Orthopedic Surgery arthritis ?

A. bacili

B. Haemophilusinfluenzae

C. 13-haemolytic streptococci

D. Streptococcus pneumoniae

E. Staphylococcus aureus

299. Which of the following conditions/factors does not predispose to gout?

A. Lesch-Nyhan syndrome

B. Psoriasis
C. Aspirin

D.Xanthine oxidase deficiency

E. Alcohol

300. A 56-year-old man is admitted with pyrexia and an acutely painful knee. On
examination, the patient is holding the joint rigidly still and is extremely reluctant to let you
manipulate the joint. The knee is obviously effused and erythematous Joint aspirate is
strongly positive for white cells (75000/3 with polymorphs accounting for 90% of these.
Blood tests show a raised ureate level. The diagnosis is:

A. Osteomyelitis

B. Septic arthritis

C. Gout

D. Pseudogout

E. Monoarticular acute rheumatoid arthritis

301. A patient presents to the outpatients department following a car accident in which she
has hurt her neck. Since then she has noticed some numbness in her right shoulder and neck
pain. On examination you note weakness in shoulder abduction , elbow flexion , absent
biceps reflex, and paraesthesiaeffecting the badge area over her right shoulder . The likely
diagnosis is:

A. Spinal cord compression

B. Radiculopathy

C.Musculocutaneous -nerve injury

D. Axillary nerve injury

E. Brown- Sequard syndrome

302. Osteoblastic bone secondaries are seen in this tumour :

A. Renal cell carcinoma

B. Non- Hodgkin's lymphoma

C. Follicular thyroid carcinoma

D. Prostatic adenocarcinoma
303. A 74 year old man tripped on a step. He fell down and is unable to stand up. He has
pain in the right hip. The whole right lower limb is abducted and externally rotated . The
most likely diagnosis is:

A. Fracture of the neck of right femur.

B. Posterior hip dislocation.

C.Central hip dislocation.

D. Fracture of the superior pubic ramus.

E. Fracture of the shaft of femur

304.Which one of the following fractures is known to cause bone necrosis ?

A. Fracture of scaphoid bone.

b. Fracture of the shaft of a metacarpal bone.

C. Fracture of shaft of humerus.

D. Fracture of inferior pubic ramus

E. Fracture of shaft of femur

305. Internal fixation is indicated for all the following fractures , except:

a. Extra-capsular fracture of neck of femur.

b. Ankle fractures

C. Supracondylar fracture of the femur with popliteal artery injury.

D. Fracture of the shaft of femur in an adult.

E. Compound fracture of the tibia.

306. Fat embolism is most likely to follow fracture of:

A. Ribs.

B. Shaft of femur.

c.Maxilla
d.Tarsal bones.

E. Clavicle.

307. About osteosarcoma all the following statements are true, except:

A. This is the second common primary malignancy of bones .

B. The peak age is in childhood .

C. Paget's disease is a predisposing factor.

D. It arises from the metaphysis of the growing end of a long bone.

E. The commonest site is the lower end of femur.

308. About osteosarcoma , one statement only is true:

A. The tumor is purely osteolytic .

B. The tumor does not invade the neighboring soft tissues

C.Hematogenous spread goes mainly to the liver.

D. Pain is the main symptom .

E. Alfa fetoprotein is elevated .

309. Regarding bone metastasis, pick up the wrong statement:

A. Patient can present with polyuria and depression

B. Pathological fractures of the femur are treated by amputation

C. The commonest primary is cancer breast

D. Prostate carcinoma gives osteoblasticsecondaries

310. The following statements about osteochondroma (exostosis) are true except:

A. Never affects bones.

B. Can leads to mechanical block of the near joint

C.Affects the diaphysis of long bones.

D. May be associated with dwarfism.


E. Can be multiple

311. The treatment of acute septic arthritis includes the following except:

A. Broad spectrum systemic antibiotics.

B. Splint in the position of function

C. Aspiration of pus.

D. Arthrotomy and drainage.

E. Local corticosteroids injection in the affected joint.

312. Generalized osteitisfibrosacystica is characterized by the following features, except:

A. Hypo-parathyroidism .

B. Diffuse rarefaction and decalcification of bones .

C. Multiple cysts of the bones.

D.Urinary symptoms due to renal calculi.

E. Pathological fracture of the affected bones.

313. The most characteristic feature of Volkmann's contracture is :

A. Wrist drop.

B. Sensory loss on the medial aspect of the forearm.

C.Extension of the metacarpo -phalangeal joints with flexion of the inter- phalangeal joints.

D.Wasting of the forearm extensor muscles .

E. Skin trophic changes

314. Complications of fracture tibia include all except:

A. Leg shortening.

B. Compartment syndrome

C. Delayed union
D. tissue injury.

E. Fat embolism

315. The nerve commonly injured with fracture neck of humerus is :

a. Axillary nerve.

B. Subscapular nerve.

C. Nerve to latismusdorsi .

D. Long thoracic nerve of bell

E. Radial nerve.

316. The incorrect statement concerning chronic osteomyelitis is :

A. Usually following inadequately treated acute attack.

B. Can be complicated by pyogenic septic arthritis.

C. Can leads to pathological fracture .

D.Sequestrum is a dead separated pieces of bone.

E. There is subperiosteal new bone formation.

317. Regarding healing of bone fractures of the following conditions are true, except:

A. Children better than in adults.

B. Transverse better than in oblique fractures.

C.Impacted better than in distracted fractures.

D. Internal better than in external fixation.

318. Regarding supra-condylar fracture of the humerus all are true except:

A. 50% of the fracture is of the green stick .

B. Malunion can lead to a cubitusverus deformity.

C. Brachial artery injury is a famous complication.


D. commonly seen among children.

E. Usually treated by a full arm plaster cast

319. The most common site of Ewing's sarcoma is:

A. Ribs.

B. Femur .

C. Tibia .

D. Radius

320. in stage of primary callus in bone healing the reaction is:

A. Acidic.

B. Alkaline.

C. Neutral

D. of the above.

321.Causes of gangrene after fracture in a limb do not include:

A. Direct crushing of the tissues

b. Injury to the main vessels

C. Tight plasters

D. Septic infection

E. Clostridial infection

322. The following are parts if displacement of Colle's fracture except:

A. Dorsal shift.

B. Dorsal

C. Radial shift.

D. Radial tilt.
E. Rotational displacement.

323. A 30 years old woman has posterior pelvic fracture, tachycardia and hypotension
responding poorly to volume replacement . US revealed free intraperitoneal bleeding and a
pelvic hematoma. The appropriate management is:

A. Application of medical antishock

B. External fixation to stabilize thearteries pelvis .

C. Laparotomy and pelvic packing measures.

D. Laparotomy and ligation of iliac

324.If an unstable hip is detected at birth the best management is:

a. Do nothing and re-examine every six months as only a minority of hips develop into a
persistent dislocation .

B. Use a splint to keep the hip joint in ° flexion and adduction.

C. Use a splint to keep the hip joint in 90flexion and abduction.

D. Advise operative stabilization

325.Which of the following is involved in the blood supply of the adult femoral head?

A. Artery of the ligamentumteres .

B. Retinacular branches of the medial circumflex femoral artery

C. Capsular branches of the superior gluteal artery

D. All are true

326. Radial nerve palsy may occur in fracture of humerus involving :

A. Surgical neck.

B. Shaft.

C. Lower end.

D. At all of the above locations


327. Regardingenchondroma all of the following are true EXCEPT:

A. It usually extends to an epiphyseal ( sub_chondral location)

B. Onion peel appearance is a characteristics x ray finding

C. Clinical & radiographic picture may mimic acute osteomylilis .

D. It is a chondrogenic bone tumor

E. It is most common in children under the age of the 4 yrs

328.A 39 years old male presents in the emergency room after a high speed motor vehicle
accident , the patient has been intubated once admitted and is on assisted ventilation , he is
unconscious , physical examination reveals a distended abdomen, and initial screening ray
reveals a displaced fracture of the pelvic ring , the initial evaluation should include :

A. Fluid resuscitation and establishment of venous access .

B. Through physical examination , including evaluation of the urinary bladder and


gastrointestinal tract.

C. Emergency application of external fixation.

D. CT of the abdomen.

329.Immobilization is not required in fractures involving :

A. Scapula .

B. Wings of ilium

C. Ribs.

D. Proximal humerus in elderly .

E. All of the above

330.Following a motor car accident a patient was found to have fracture shaft femur. Which
of the following is a sure physical finding of concomitant arterial Injury?

a.Large thigh hematoma

B. Pulsating thigh swelling

C. Extensive echymosis over the fracture site


D. Absent dorsalispedis pulse

331.In contrast to closed reduction, open reduction of fracture:

A. Produces a shorter healing time.

B. Decreases trauma to the fracture site.

C. Reduces the risk of infection.

D. Requires longer period of immobilization.

E. None of the above

332.In tuberculosis of the bone :

A. The local reaction is characterized by extensive new bone formation.

b. The metaphysis is the commonest site involved .

C. The infection is usually secondary to a distant focus of the disease.

d. Extension of the bone abscess into the joint is common

333.When performing a four compartment fasciotomy for compartment syndrome in leg,


radial and lateral incisions are created. Which of the following compartments is opened
through the medial incision?

A. Anterior compartment

B. Deep posterior compartment

C. Peroneal compartment

D. None of the above

334.Which of the following regarding the Gustillo classification of fractures is FALSE?

A. It applies only to the soft tissues.

B. It relies primarily on the length of any laceration

C. It takes account of whether or not there is soft-tissue cover of fractured bone.


D. It takes account of contamination.

335.Of the following statements regarding the treatment of fractures are true?

A. All fractures should be reduced

B. Relative stability means that some movement at the fracture site is going to occur.

C. Absolute stability is obtained by getting exact reduction and then compressing the
fragments of the fracture together.

D. Absolute stability should not attempted , in achieving itthe blood supply to the bone will
be compromised

336.Which of the following orthopedic screws are most commonly used to secure a distal
bone fragment to a more proximal fragment?

a.Cortical screws

B. Cancellous screws

C. Lag screws ( malleolar lag screws)

D. Thompson screws

337. pupic ramus fractures are most often associated with :

A. Injuries to the urethra

B. Injuries to a hollow viscus

c. Sacral fractures

D. Acetabular fracture

338.A largo acute tear in the medial meniscus in a young athlete is best treated by :

A. Immobilization and anti- inflammatory agents

B. Repair of the meniscus

C. Resection of the meniscus

D. Resection of the meniscus and replacement with allograft


339.A 60-year-old man hit by a pickup truck and brought to the emergency department with
a blood pressure of 70/0 mm Hg. Peritoneal lavage showed no blood in the abdomen. The
blood pressure is elevated to 85 systolic following the administration of 2 L of Ringer's
lactate. An x-ray showed a pelvic fracture. What is the next step in management?

A. Exploratory laparotomy with packing of the pelvis

B. CT scan of the pelvis

C. External fixation of the pelvis

D. Open reduction and internal fixation (ORIF) of the pelvis

E. Exploratory laparotomy with bilateral ligation of the internal iliac arteries

340.A 65-year-old man is brought to the hospital after being hit by a car . His blood pressure
is 150/90 mm Hg, and pulse is 120 bpm. There is deformity just below the left knee and no
distal pulses palpable in that leg. Plain films show proximal tibia and fibula fractures . What
is the next step in management?

A. Operative intervention to restore flow with an arterial shunt

B. Angiography

C. Doppler ultrasound

D. Operative reduction and internal fixation

E. Heparinization

341.A 43-year -old woman is thrown from a car following a car crash. She presents to the
emergency department with a fracture of the pelvis. Her blood pressure is 80/60 mm Hg,
pulse is 110 bpm, and respiratory rate is 26 breaths per minute. Bright red blood is found on
rectal examination and bony fragments can be palpated through the rectal wall. The patient
remains hypotensive despite 3 L of Ringer's lactate and 2 U of type-specific blood. What is
the most important step in management ?

A. Exploratory laparotomy and colostomy

B. External fixation of the pelvic fracture

C. PASG

D. Fresh- frozen plasma

E. Wiring of symphysis pubis


342.A 47-year-old man awakens with low back pain after a weekend of gardening. He recalls
no specific incident of trauma and has never had back pain before. There is no radiation of
the pain and no disturbance of normal bowel or bladder function. The ROM or the low back
is painful and restricted in all planes, and there is para-spinal tenderness from L2 to L5 on
the right. Scoliosis and kyphosis are absent. Findings on straight-leg- raising test are
negative, reflexes are active and equal, and the patient can walk on his heels and toes.
Findings on x-rays of the lumbar spine are normal. Which is the best treatment?

A. Bed rest for 48 hours, anti-inflammatory agents, heat to the low back, and nonnarcotic
analgesics.

B. Bed rest for 7-10 days, heat to the lower back, anti-inflammatory agents, muscle
relaxants, and analgesics

C. Hospitalization for pelvic traction, physical therapy, anti-inflammatory agents,


intramuscularly analgesics, and muscle relaxants

D. Immediate magnetic resonance image (MRI) for the lumbar spine

E. Lumbar puncture

343.A 40-year-old woman was involved in a car crash. She was unconscious for 5 minutes .
X-ray revealed a depressed fracture in the frontal region. Which of the following statements
is true of skull fracture ?

A. It always requires surgical exploration .

B. It is compound if multiple .

C. It requires burr holes if compound .

D. In the anterior cranial fossa , it may produce rhinorrhea .

E. It requires steroid administration

344. A30 - year old man sustained a pelvic fracture with a large pelvic hematoma . Rectal
examination reveals a large laceration in the rectal wall and a non palpable prostate . His
vital signs have stabilized with multiple transfusions . This patient requires which of the
following ?

A. Resuscitation , blood transfusions , external fixation , and exploratory laparotomy .

B. Resuscitation , angiography , embolization of the pelvic bleeders , exploratory laparotomy

C. Resuscitation , broad -spectrum antibiotics , retrograde cystourethrogram , CT of


abdomen and pelvis , suprapubiccystostomy , and diverting colostomy .
D. Exploratory laparotomy urinary diversion sigmoid colostomy , presacral drainage ,
anddebridement of the rectal wall .

E. ORIF of pelvic fracture by a posterior approach , colostomy , and suprapubiccystostomy .

345.Which of the following is the most common secondary cause of haemarthrosis of the
knee?

A. Meniscal tear

B. Anterior cruciate ligament injury

c. Osteophyte fracture

D. Posterior cruciate ligament injury

E. Patella dislocation

F. All are the wrong

346.According to the current National Institute for Health and Clinical Excellence guidelines,
all of the following are indications for use of bisphosphonates in the treatment of
osteoporosls, except:

A. Any patlent with a fracture over the age of 65

B. Any patient with a fracture aged 65-75 and a T score less than -2,5

C. Any patient with a fracture aged <65 and a T score less than -3.0

D. Any patient taking high-dose systemic steroids for more than 3 months with a T score less
than -1.3

E. Any smoker with a fracture aged <65 and a T score less than -2.5

347.About osteosarcoma , all the following statements are true, except?

A. Plain X-ray shows Codman's triangle.

B. Definitive diagnosis rests on a combination of clinical and radiographic findings (neither


sunray nor codman triangle is pathognomonic of osteosarcoma)

C. Wide excision is the main treatment of early cases.

D. Limb salvage is possible in early cases.

E. Chemotherapy improves outcome of treatment


348.About giant cell tumors of bone, all the following statements are true, except:

A. It may be multi-locular.

B. The main symptom is pain.

C. It can cause pathological fracture.

D. The cut section of the tumor is reddish brown.

E. Treatment is by wide resection with a safety margin , followed by chemotherapy.

349. Regarding fracture Pelvis all are true except:

A. It is unstable, if there is disruption of the true pelvic ring .

B. Commonly associated with visceral injury.

C. Open book type is due to vertical shear trauma .

D. Always treated with skeletal traction.

E. usually due to road-traffic accidents.

350.Intertrochanteric fracture neck femur:

A. Usually complicated by a vascular necrosis.

B. Hip hemi arthroplasty is the treatment of choice.

C. Intra medullary nail can be a line of treatment.

D. is an extra-capsular fracture.

E. The limb is usually adducted and externally rotated

88. Urgent surgery Is Indicated for patients with prolapsed lumbar disc when presented with

a. Severe back pain not responding to medications

b. Persistent numbness of lower limbs


c. Foot drop and urinary retention

* Questions (89 &90) : A 57-year-old woman is referred to you for evaluation of difficulty
with ambulation. Her chief complaint is weakness of her left leg that has been slowly
progressive over the last 6 months . On neurologic examination, her mental status and
cranial nerve findings are within normal limits. She has marked (grade ) weakness of both
her left leg and arm. On her left side, she has diminished sensation to light touch and
vibration below the dermatome. Sensation to pinprick and temperature are severely
diminished on the right side below approximately the C8 dermatome. Her deep tendon
reflexes and muscle tone are increased on the left

89. This pattern of neurologic deficits is which of the following?

A. Spondylolisthesis

b. Brown-Sequard syndrome

C. Central cord syndrome

D. Guillain -Barré syndrome

E. Poliomyelitis

90. This pattern of neurologic deficits is explained by injury to the spinal cord with damage
to which of the following?

A. Anterior hom cells

B. Peripheral neuropathy

C. Central cord

D. Right half (right hemicord)

E. Left half (left hericord)

91. The commonest cause of bilateral brachial neuralgia is :

A .Cervical spondylosis.

b. Subacromial bursitis.

C. Supraspinatus tendinitis.

D. Spinal tumor

E. Angina pectoris.
92. An intervertebral disc is composed of :

A. A central mass of loose connective tissue.

B. A peripheral ring of tough fibrous tissue.

C. Two plates of hyaline cartilage.

D. All of the above.

E. A and B only.

93. In the treatment of kyphosis , spinal osteotomy is indicated only in :

A. Sheuermann's disease .

B. Senile kyphosis.

C. Pott's disease

D. Ankylosing spondylitis.

E.Rommel's disease

94. Scoliosis may be complicated , by the following except:

A. Back pain.

B. Reduced vital capacity of the lungs.

c. Empyema.

D. Corpulmonale .

E. Paraplegia.

95. The most common primary tumor of the spine is :

A. Osteoclastoma

B. Chondroma .

C. Bone sarcoma .
d. Fibrasarcoma .

E. Multiple myeloma .

96. As the emergency department foundation year 2 doctor on call, you are asked to
perform a lumbar puncture . The mid -point at the level of the iliac crests is identified as
your point of insertion of the puncture needle . Which intervertebral space does this
represent ?

a. L 1/L 2

b.L2/L3

C.L3/L4

D. L4/L5

E. L5/ S1

97.White explaining the procedure of a lumbar puncture to a junior colleague , you are
asked about the layers through with the lumbar puncture needle must pass before reaching
the area containing cerebrospinal fluid . After piercing the skin and subcutaneous tissues ,
which anatomical structure would be traversed next during a tap?

A. Interspinous ligament

B. Supraspinous ligament

C. Dura mater

D. Ligamentumflavum

E.Epidural space

42. A 23-year- old woman complains of progressive loss of vision and papilledema.
Investigations show normal findings on CT scan. A lumbar puncture shows marked elevation
of pressure. What is the most likely diagnosis?

A. Pseudotumorcerebri

b. Corpus cavernous thrombosis

C. Cavernous sinus thrombosis


D. Retinoblastoma

E. Chordoma

43. Regarding Cerebrospinal fluid all are true EXCEPT:

A. Normally has a lower protein content than plasma

B. Flows between the third and fourth ventricles via the foramen of Monro

C. is sterile

D. Is produced at a rate of 0.5

E. absorbed by the arachnoid granulations

44. Froin's syndrome consists of the following features except:

A. High intra-spinal pressure.

B. Yellow coloration of the CSF (Xanthochromia).

C. Spontaneous coagulation.

D. increased protein content.

E. No increase in cells.

45. The causes of internal hydrocephalus do not include:

A. Excessive secretion of CSF by the choroid plexuses.

B. Congenital stervosis of the aqueduct.

C. Midline brain tumors.

D. Arnold-Chiari malformation.

E. Basal adhesions due to subarachnoid hemorrhage, meningitis or sarcoidosis.

46. All of the following can be used to treat elevated intracranial pressure except:

A. Elevate the head of the bed


B. Place a ventricular catheter

C. Mannitol bolus

D. Hypertonic saline IV

E.Glutamate antagonists Nerve Injury

47. Sympathectomy is most effective in:

a. Raynaud's disease.

b. Buerger's disease

c. Acrocyanosis.

d. Scleroderma. .

e. Causalgia.

48. Horner's syndrome is characterized by the following except:

a. Ptosis of the upper eyelid.

b. Constriction of the pupil (miosis).

c . Enophthalmos.

d. Flushing of the affected side of face.

e. Excessive sweating of the same side of face.

49. Following repair of a completely transected peripheral nerve, regeneration usually


proceeds at the daily rate of:

a. 1 mm.

b. 5 mm.

C. 1cm

d. 1inch

50. The signs of ulnar nerve injury at the wrist include the following except:

a. Ulnar claw-hand deformity.


b. Flattening of hypothenar eminence and hollowing of interosseous spaces.

c. Positive Froment's sign.

d. Failure to grip a sheet paper between two extended fingers.

e. Weakness of hand grasp and of flexion of wrist.

51. Division of the median nerve above the wrist manifests itself clinically by the following
signs except :

a. "Ape- hand" deformity.

b. Loss of opposition of thumb to little finger .

c. Preservation of pronation of forearm.

d. Pointing index during clasping the hands.

e. Anaesthesia over palmar aspect of radial side of hand and of lateral 3 ^ (1/2) fingers.

52. Delayed ulnar neuritis is due to:

a. Fractures and dislocations in the elbow region.

b. Wounds of the arm, forearm and wrist.

c. Cubitus valgus deformity.

d. Leprosy.

e. Neurofibromatosis

f. TB

53. A patient presents with numbness in the 1st, 2nd and 3rd toes. The nerves contributing
to the numbness include :

a. Medial plantar nerve.

b. Lateral planter nerve.

C. Superficial peroneal nerve.

d. Sural nerve.
54. Ape hand deformity is due to paralysis of:

A Extensors policislongus

B. Opponenspolicis

C. Adductor policis

D. All of the above

55. Clawing of any finger necessitates paralysis of the following muscle :

a. Dorsal interossei

b. Palmar interossel

c. Lumbricals

d- All of the above

56. As regards Expectant treatment of peripheral nerve injury all are correct except :

a-indicated in all nerve injuries

b- Only indicated in closed types

c - Include massage of active exercises

d- Usually continue for 6 months

57. All of the following are true about carpal tunnel syndrome except:

a. It is caused by entrapment of the median nerve at the wrist.

b. Flexion at the wrist bringing on symptoms known as Tinel's sign

c. Risk factors include hyperthyroidism and pregnancy

d. Patients often wake with pain and paraesthesia in the thumb index and middle fingers .

58. The sympathetic nervous system arises from:

a. Cranial nerves III, VII, IX, and X


b. Cranial nerves II, IV, Vand VII

c. The thoracolumbar spinal segments

d. Spinal segments S2, S3 and S4

59. A 36- year-old man developed neck and left arm pain. He noted paresthesia in the left
Index and middle fingers . He was found to have weakness of the left triceps muscle and a
diminished left triceps jerk. His left- sided disc herniation is most likely to be at:

a. C3-C4 .

b. C4-C5.

c. C5-C6

d. C6- C7

60. In radial nerve crush Injury, the most appropriate treatment is:

a. Immediate exploration and repair

B. EMG two weeks after injury and surgical intervention is indicated the nerve conduction is
not improved.

C. Surgical intervention in 12 weeks if no improvement occurs

D. None of the above.

61. Following nerve injury the worse prognosis is for:

a. Pure motor nerve.

b. Mixed nerve.

c . Pure sensory nerve.

d. All of the above.

62. A 55-year-old female presents with 3-years history of severe lancinating pain extending
from left ear to her maxillary areaPain is triggered by chewing and brushing teeth. She was
treated by otolaryngologist for sinus infection a year ago and undergone multiple dental
work and teeth extraction with transient or no improvement. The most likely diagnosis is:
a.Maxillary sinusitis

B. Trigeminal neuralgia

C. Maxillary osteomyelitis

d. Gradenigo's syndrome

E. Otitis media

63. The neurovascular structure most commonly injured as a result of an anterior


dislocation of the shoulder is the:

A. Musculocutaneous nerve .

B. Axillary nerve .

C. Axillary artery .

D. Median nerve .

64. Sciatic nerve injury manifests itself by the following signs except

a. Paralysis of all muscles below the knee.

b. Drop foot and clawing of the toes .

c. Anesthesia of whole leg and foot.

D. Trophic changes in sole of foot and toes.

E. Causalgia if the nerve lesion is partial.

65. An elderly male presented with a firm painless movable subcutaneous mass in his right
side. The mass was excised and histological examination revealed that the mass was
composed of palisade spindle cells with a peripheral nerve at one end. The next step in the
management of this patient is to :

A. Reassure the patient that the surgery was curative

b. Re-excise the area for wider margins.

C. Give prophylactic radiotherapy.

D. Re-examine the patient caffe-au-lait patches.

E. Order investigations for nerve root lesions.


66. All of the following are true regarding congenital talipesequinovarus except:

A. There is adduction at the tarso-metatarsal joints

B. It is a relatively common deformity

C. It is more common in females

D. There is inversion of the calcaneus and the navicular on the tibia

67. The incorrect statement about acoustic neuroma is that it:

a. Arises from the neurolemma sheath of the acoustic nerve

b. May be bilateral and associated with "cafe -au-lait patches.

c . Grows slowly in the cerebellopontine angle.

d. Involves the eighth nerve only.

e. May cause cerebral and pyramidal signs.

68. of the following are true about astrocytomas except:

a. They are gliomas.

b. They can present with seizures .

C. The cell of origin is the oligodendrocyte.

d. They are radiosensitive.

69. All of the following are true about meningiomas except:

a. They are usually benign.

b.80 % are supra- tentorial.

c. They can cause cytotoxic brain edema .

d. The 10 -year recurrence rate after complete excision is 10%.

e. They are often slow-growing .


70. The most common malignant tumor of the brain in children is:

a. Ganglioneuroma .

b. Medulloblastoma .

c. Neuroblastoma

d. Glioblastomamultiforme

151. As regards causalgia all are correct except:

a-Commonly occurs in ulnar nerve injury

b- due to partial injury

C- a constant pain sensation in area supplied by Injured nerve

d- Is treated by central pain killers in severe cases

152. As regards types of nerve injury all are correct except:

a- Axontemesis has the best prognosis

b- Neurontemesis is complete sectioning of nerve

c - No wallerian degeneration occurs in neurapraxia

d-All of the above

153. As regards Expectant treatment of peripheral nerve injury all are correct except:

a-Is indicated in all nerve injuries

b- Only indicated in closed types

c-Include massage of active exercises

d-Usually continue for 6 months

154. As regards Klumpke's paralysis all are correct except:

a-Means injury to upper trunk of brachial plexus


b- The main presentation is complete clawing

c- May be associated with Horner's syndrome

d- Most common cause is complicated breech delivery

155. In cases of median nerve paralysis the following muscle is expected to be paralyzed :

a- Extensor pollicislongus

b- Adductor pollicis

c- Opponenspolicis

d- Third lumbrical

156. Pseudomotor affection means:

a-Loss of reflexes

b-Loss of voluntary movements

c - Loss of involuntary activitiesd

d-Loss of gland activities (anhydrosis )

157. Ulnar paradox occurs in ulnar injury:

a- At the wrist

b- Above the elbow

c- Both of them

d-None of them

158. Injury of radial nerve in spiral groove differs from injury in axilla in that:

a- In injury in axilla there is loss of supination

b- Injuries in spiral groove spares nerve supply to long head of triceps


c-extensors of wrist are not affected in injuries in spiral groove

d- All of the above

159. Ape hand deformity is due to paralysis of:

a- Extensors policislongus

b- Opponenspolicis

C- Adductor policis

d-All of the above

160. Clawing of any finger necessitates paralysis of the following muscle:

a. Dorsal interossei

b. Palmar interossei

C. Lumbricals

d. All of the above

161. The most effective treatment of persistent causalgia is:

a. Sympatholytics

b. Physiotherapy.

c. Sympathectomy

d. None of the above .

162. Causes of carpal tunnel syndrome include all of the following except:

a. Pregnancy

b. Myxedema.

c. Cervical rip.

d. Rheumatoid arthritis .

e. After colle's fracture.

f. Often associated with vascular disorder.


163. A patient presents with numbness in the 1st, 2nd and 3rd toes. The nerves contributing
to the numbness include:

a. Medial plantar nerve.

b. Lateral planter nerve.

c. Superficial peroneal nerve.

d. Sural nerve.

164. Foot drop may result from:

a. Lumbar disc prolapse.

b. Fracture neck of fibula.

C. Para-sagittal meningioma.

d. Peripheral neuropathy.

e. All of the above.

f. None of the above.

165. Early signs in compartmental syndrome of the anterior compartment of leg is :

a. Pressure feling in limbs.

b. Absent distal pulses or firm calf.

C. Pain on passive stretching the affected muscles.

d. Numbness and paraesthesia in the web space between the 1 ^ (st) and toes ( deep pain).

166. Delayed ulnar neuritis is due to:

a. Fractures and dislocations in the elbow region .

b. Wounds of the arm, forearm and wrist.

c. Cubitus valgus deformity.

d. Leprosy

e. Neurofibromatosis.
f. TB

167. The signs of ulnar nerve injury at the wrist include the following except:

a. Ulnar -hand deformity.

b . Flattening of hypothenar eminence and hollowing of interosseous spaces.

c. Positive Froment's sign.

d. Failure to grip a sheet paper between two extended fingers.

e. Weakness of hand grasp and of flexion of wrist.

168. Division of the median nerve above the wrist manifests itself clinically by the following
signs except:

a. 'Ape- hand" deformity.

b. loss of opposition of thumb to little finger.

c.Preservation of pronation of forearm .

d. Pointing index during clasping the hands.

e. Anaesthesia over palmar aspect of radial side of hand and of lateral 31/2 fingers.

169. Median nerve injury at the wrist results in the following except:

a.Loss of sensation over the palmar aspect of the lateral three and half digits.

b. Inability to oppose the thumb to the other fingers

c. Inability to the terminal phalanx of the thumb.

d. Ape-thumb deformity.

e . Wasting of the thenar eminence.

170. Horner's syndrome is characterized by the following except:

a.Plosis of the upper eyelid

b.Constriction of the pupil (miosis).

c.Enophthalmos.
d.Flushing of the affected side of face.

e.Excessive sweating of the same side of face.

171. Following repair of a completely transected peripheral nerve, regeneration usually


proceeds at the daily rate of:

a. 0.1 mm.

b. 1 mm.

C. 5 mm

d. 1cm

e. 1inch

172. Sympathectomy is most effective in :

a Raynaud's disease .

b. Buerger's disease.

c. Acrocyanosis.

d. Scleroderma.

e .Causalgia

173. Concerning fracture-dislocations of the spine, the wrong statement is that they:

a. Occur most often in the lower cervical region.

b. Result from excessive flexion -rotation injury of the spine.

c. Are associated with rupture of the " posterior ligament complex".

d. Consist of forward dislocation of the upper vertebra and wedging or crushing of the lower
vertebra.

e. Are commonly associated with paraplegia.

174. The most valuable diagnostic method in hydrocephalus is:

a. Plain X- ray examination .


b. Examination of CSF

c. Dye test

d. CT scanning .

e. Ventriculography .

175. Depressed fractures of the skull are characterized by the following except:

a. Are often compound

b. May involve the base of the skull

c. May be associated with profuse bleeding, leakage of CSF or protrusion of brain matter.

d. Always require urgent operation.

e. May be followed by post-traumatic epilepsy.

176. The signs of fracture of the anterior cranial fossa includeexcept:

a . Epistaxis .

b. Cerbrospinalrhinorrhoea

c. Subconjunctival hemorrhage .

d. Bleeding from the ear.

e.Injury to the first six cranial nerves .

‫تجميع اسئلة الرابط االخير‬

Reset exam of orthopedic surgery for the 5 th grade

(1) Which of the followings is not a predispositional factor in DDH development ?

A- Prenatalbreechposition

B- Ligamentous hyperlaxity

C- First child

E- Post natal hip position


(2)- in DDH the acetabulum is said to be dysplastic if it is ?

A- less than 40

B- more than 40

C- between 30 – 27 D- less than 20

E – more than 20

(3)- Which of the following lines cross the triradiate cartilage in DDH ?

A- Hilgenreiners line B- Perkins line

C- Shanton line

D- Calves line

E – Von Rosen line

(4) The following statements are true about Perthes disease except?

A- It is the avascular necrosis of the femoral head . B- Its main eitiology still unknown

C- It is common in the first year of life

D- It is often present as a painless limb

E - Small femoral epiphysis is an early sign seen on X-Ray.

(5) Which of the following tumors is an epiphyseal tumor ?

A- Giant cell tumor

B- osteosarcoma

C- anurysmal bone cyst D- Ewings sarcoma

E – Chondrosarcoma
(6) What is your diagnosis?

A- Chondroblastoma

B- Osteoid osteoma

C- Enchondroma

D- Osteochondroma

E- Osteosarcoma

(7) an open fracture with a vascular compromise is a Gustillo Anderson type

A- 1

B- 2

C- 3A

D-3B

E -3C

(8) Montague fracture with a posterior radial head dislocation and ulnar proximal shaft
fracture is Bado type ?

A – TYPE 1

B- TYPE 2

C- TYPE 3

D- TYPE 4

E – NON OF THE ABOVE

(9) Which of the following statements is wrong regarding Colles fracture?

A-it involve the distal 2.5 cm of the metaphysic of the radius with dorsal angulation

B- Frykman classification is the most commonly used classification to describe Colles


fractures
C- The median nerve is the most commonly injured and primarily neuropraxic

D- change in the radial inclination angle less than 5 degree is acceptable.

E - palmar tilt less than 10 degree is not acceptable

(10) Which of the following statements is wrong related to Garden classification of femoral
neck fractures?

A- TYPE 1 carries the lowest risk of femoral head AVN

B- TYPE 2 the fracture line is complete and the inferior cortex is fractured.

C- Type 3 there is a mild displacement and it carries the highest risk of AVN???

D- TYPE 4 the fracture is complete and displaced significantly

E- Garden classification gives a good prognostic value of the femoral head situation

(11) Which of the followings is the most common malignant soft tissue tumor ?

A- malignant fibrous histocytoma

B – fibrosarcoma

C- liposarcoma

D- Ewings sarcoma

E – malignant schwannoma

(12) Which of the following statements is wrong regarding septic arthritis?

A – hematogenous septic arthritis is common in pediatric population

B- Staphyloccocus aureus is the most common pathogene.

C- Arthroscopic drainage is appropriate and safe treatment of every joint in the body.

D- arthrotomy is more preferred in pediatric population.

E – septic arthritis is not an orthopedic emergency and it can be delayed until the condition
of the patient is deteriorated.
(13) Which of the following joints is the most commonly involved in septic arthritis?

A- the knee joint

B- the hip joint

C- the shoulder joint

D- the elbow joint

E – the ankle joint

(14)- Which of the following statements is wrong regarding acute osteomyelitis ?

A- Hematogenous infection occurs mostly in the metaphyseal area.

B- X- Ray shows some sequestrum in the metaphyseal area.

C- ESR is a good indicator of the activity of the infection status.

D- in association with osteomyelitis Septic arthritis may occur in pediatric population due to
a common metaphyseal epiphyseal blood supply .

E – it is associated in most cases with severe pain swelling and guarded movements.

(15) patient with sickle cell anemia are prone mostly to which type of osteomyelitis?

A- H. influenza osteomyelitis

B- Streptococcus osteomyelitis

C- salmonella osteomyelitis

D- E. coli osteomyelitis

E – Pseudomonas osteomyelitis

(16) what is your diagnosis ?


A- L4-L5 Spondylolysis

B- L4-L5 Retrolisthesis

C- L5-S1 Spinal stenosis

D- L4-L5 Spondylolisthesis

E- L4-L5-S1 spondyloarthropathy

(17) Which of the following structure is the corresponding structure of the body of the
Scottish dog of the spine?

A- The lamina

B- the pedicle

C- the superior articular process

D- the inferior articular process

E – the spinous process

(18) Which of the following is not a rotator cuff muscle of the shoulder ?

A- Supraspinatus

B- infraspinatus

C- teres minor

D- teres major

E- subscapularis
(19) Which of the following statements is true regarding DDH:

A- Ortolani test can not be used in infants due to soft tissue changes

B- Galeazzi test is not a specific test specially in bilateral hip dislocation

C- In Nelatons test great trochanter should be located below the drown imaginary line.

D- Von Rosen line is a line drown from the great trochanter passes through the SIAS to the
umbilicus.

E -Barlow sign is a clunk sound heared while the hip is reduced into the acetabulum.

(20 ) Which of the following statements is wrong regarding ankylosing spondylitis?

A- It is characterized by inflammation of articular and paraarticular structures.

B- The basic pathologic lesion of ankylosing spondylitis occurs at the entheses, which are
sites of attachment to bone of ligaments, tendons, and joint capsules.

C- Approximately 90-95% of patients with ankylosing spondylitis have the tissue antigen
human leukocyte antigen B27 (HLA-B27).

D- Life expectancy is unchanged , Most patients are able to maintain a normal lifestyle.

E- One of the earliest radiological manifestation is the bamboo spine.

(21)- The most common complication of pelvis fracture?

A . haemorrhagic shock

b. injury of the uterus

c. injury of urinary bladder

d. septic shock

E. rectal injury

(22) the most common complication of subcapital neck of femur fractures ?

A. osteomyelitis
B.coxa varus

C. sciatic nerve injury

D. avscular necrosis

E. malunion

(23) which of the following criteria is acceptable in tibial platue fracture ?

A. if displacement of articular surface more than 2mm

B . if displacement of articular surface less than 2mm??

C. if fragment is rotated more than 45 degrees

D- If split is more than 1 cm

E- non of the above is true

(24) the most universally accepted method of classifying radiographic osteoarthritis, uses
the following radiographic features except :

A. Joint space narrowing

B .Osteophytes

C. Loose Body

D. Subchondral sclerosis

E. Subchondral cysts

(25) The classic initial site of involvement of ankylosing spondylitis is,

A. Wrist Joint

B. Atlanto Axial Joint

C.The sacroiliac joint

D. Ankle Joint

E. Hip joint
(26)- which of the following is wrong regarding osteoid osteoma?

A- it is an osteoid producing tumor

B- it has a malignant transformation of 5% to osteosarcoma

C- it causes pain at night relieved by aspirin

D- its nidus consists of high level of prostaglandin E2

E - it may regress with time

1. Muscle innervated by peroneal nerve ->> short head of biceps

2. All pass through carpal tunnel except >>> FCR

3. METS to bone MC >> spine

4. Hallux vulgus whivh one is wrong ? (matary)

5. T/F extracapsular fracture of upper end of femur usually treated by internal fixation. ???
(matary)

6. Perthe’s disease (matary) osteochondritis of proximal femoral ...?

water >>> 7. . ‫اكتر اشي من مكونات الغضروف وزن‬

8. Trigger finger >> stenosing tenosynovitis of flexor tendon (matary q 98)

9. Tennis elbow >> (matary q 96) >> lateral epicondylitis

10. Man with pelvic fx .. blood on urethral meatus ..what can Complicate his condition ? >>
floey’s

cath

11. Fx of surgical neck of humerus >> loss of sensation over deltoid

12. Male with tibia mid-shaft fx treated with IM nail presented with redness and swelling >>
best diagnostic test >> aspiration and culture
13. All arise from coracoid process of scapula except >>> long head of biceps

14. Best prognosis in nerve injury ?? >> neurpraxia

15. MC organism in human bite ??>> ekenella

16. All are late complications of fx except >>> osteitis fiborsa ossifica

17. Healing time for simple uncomplicated adult tibia fx >>>> ? 3wks .. 8wks .. 12wks ..
6wks.. 36wks

18. Compartment syndrome all true except >>> loss of pulsation is an early sign ..... can be
mistaken for DVT ???

19. Most important factor in Osteoarthritis >>> aging.. obesity .. fx???

20. Sex hormone that is important for bone mass >>> estrogen

21. Osteogenesis imperfecta >>> type 1 collagen

‫ عل االرجح بتعتمد برضو على الخيارات الثانية‬All are complications of plaster except >>> malunion .22

23. ACL reconstruction MC used for graft >>> semimembranous and gracilis

24. Supracondylar fx of humerus , can’t flex index or ( ) of thumb what is the injured nerve??
?? ulnar ... radial... median... pin.. ain ???

25. Tennel sign ???? >>>>> nerve injury,, nerve atrophy ,,, nerve recovery ???

26. Mid tibia fx .. intramedullary nail most common (complication???) >>> infection ...
malunion...

non-union.. compartment syndrome

27. Posterior approach to knee , you cut medial to the s. saphenous vein in order to avoid
injury of

which structure ?? >>>> medial cutaneous sural n..... common peroneal nerve... tibial
nerve..?

28. Referred knee pain from hip which nerve ?? obturator

29. RA question T/F matary ..

30. MM matary

31. Lateral dislocation of patella ??? >>> injury of medial patellofemoral ligament .... Injury
to quadriceps lig ..... injury to medial patellatibial lig ???
Orthopedics Exam 2009

1. Sciatic nerve supply all of the following except:

Answer: Sartorius muscle

2. Popliteal fossa contains all of the following except:

Answer : Superficial peroneal nerve

3. Most common fracture of the carpal bone is:

Answer: scaphoid bone

4. Osteoid osteoma:

oval depression surrounded by sciatic nerve

5.Radial nerve palsy associated with following injury to humerus:

Answer : Shaft

6. Spiral fracture caused by:

Answer :Twist

7. One of the following will cause confusion with giant cell tumor:

Answer: brown tumor of parathyroid

8. Most important factor in fracture healing is:

Answer : Immobilization

9. All of the following cause locked knee except:

Answer : infrapatellar bursa


10. Calcium absorption in intestine:

Answer : Vit D

11. Prognostic factor in angular fracture healing: ?????????????????????????

12: Shoulder dislocation most likely occur:

Aswer: anteriorly

13. Shortened adducted and internally rotated lower limb:

Answer : posterior dislocation

14. Midshaft clavicle fracture:

Answer : sling for 3 weeks

15. Colle'sfracture:

Answer: most common on older

16.Prolapsed lower intervertebral disc, all of the following may result, except:

Answer : extensor plantar reflex

17. Most common joint to be dislocated:

Answer : Shoulder

18. One of the striking thing about hematogenous osteomylitis is:

Answer :the mortality


19. Most common pathogen causing osteomylitis:

Answer : S.aureus

20. Case of female with below elbow cast, suddenly no movements to the thumb, most likely
injury is:

Answer : injury to extensor hallucis longus

21. Pseudogout:

Answer : calcium pyrophosphate crystals

22. Initial sign of osteoarithritis:

Answer : sclerosis of the cartilage

23. Origin of the bone:

Answer: mesoderm

24. The nerve affected in carpal tunnel syndrome:

Answer : median nerve

25. Colle's fracture, all except:

Answer : ulnar injury

26: green stick fracture:

Answer : fracture one cortex is fractured and other isn't

27. Brodies abscess all of the following true except:

Answer : it is a form of chronic osteomylitis


28. Dupuytrenes contracture all of the following except:

Answer : liver cirrhosis

29. Sequestra:

Answer : a piece of dead bone

30 . Tennis elbow:

Answer : inflammation of the common extensor tendon as it inserts

into the lateral epicondyle.

Q1 The sequestrum in X-ray appears:

a. Dense

b. Light

c. Isodense as surrounding bone

d. Any of the above

Q2 Osteoblastic bone secondaries commonly arise from cancer of:

a. Breast

b. Lung

c. Prostate

d. Adrenal

Q3 Tears of the meniscus of the knee result from which of the following strain:
a. Hyperextension

b. Abduction

c. Adduction d. Rotation

d. Combined flexion and rotation

Q4 The single dependable sign of early Volkmann's contracture is:

a. Cyanosis of fingers

b. Obliteration of radial pulse

c. Paralysis of flexor muscles of forearm

d. Pallor of fingers

e. Pain

Q5 Radial nerve palsy may occur in fracture of the humerus involving:

a. Surgical neck

b. Shaft

c. Lower end

d. At all of the above locations

(Or: The most common injured structure with mid-shaft humerus fracture Answer: radial
nerve injury)

Q6 Best way to preserve amputated part for re-implantation

a. Cold normal saline

b. Cold ringer lactate

c. Cooling with dry ice

d. Freezing

Q7 which of the following is the most common source of loose bodies in the knee joint:

a. Synovial Chondromatosis

b. Osteochondritis Dissecans

c. Osteophytes
d.fracture articular surface

Q8 Osteoid osteoma originates from:

a.Periosteum

b. Cortex

c. Medullary cavity

d. All of the above

Q9 Most common cause of death 3 days after hip fracture:

Answer: Fat Embolism

Q10 Which of the following statements is False:

Answer: sternoclavicular joint injuries are more common than Acromioclavicular joint
injuries

) ‫ (مش عارفين‬:Q11 Earliest laboratory finding in fat embolism is

a. Lipase

b. Cholesterol

c. Fat

d. Alkaline phosphatase

Q12 Second most important step in Treatment of long fractures:

Q13 Best way to stop bleeding before reaching hospital in a patient with open wound in leg:

e legs

Q14 Regarding the posterior dislocation of the shoulder, all are true except:

) ‫(مش متأكدين‬ non-reduction

Q15 which of the following is considered to be an indication for open reduction


‫ كان االمتحان‬general ‫وعادي النھ حكالنا ما كتب امتحان بس اعطانا امتحان من سنة قبلنا وكانوا الطالب مش وال بد‬
‫فكان‬
‫امتحانھم سھل‬

‫ ما جاب غير يمكن سؤالين من ال‬.. ‫حكالنا انھ االسئلة معظمھا من النت وانھ الزم حلينا اسئلة جراحة عظام من النت‬
mattary ‫ وسؤال من‬essential

‫سأل عن‬:

1. ‫ وحدة عندھا‬compound tibial fracture management‫ ركزي على انھ الكسر‬، compound ‫يعني‬
external fixation: ‫ فالجواب بكون‬..internal fixation ‫ لما يكون الكسر ھيك ممنوع نعمل‬.. open

2. ‫ لما فحصناھا كان‬.. ‫ بعد فترة نفخت ركبتھا وصارت توجع‬، ‫ وقعت على ركبتھا‬،‫حالة طويلة عن وحدة بتلعب تنس‬
5/5 ‫ عندھا‬fluctuant mass, no redness, no fever, full ROM, muscle power.. septic arthritis ‫بما انھ‬
‫فش حرارة يعني مش‬ ‫ يعني ما عندھا ال كسر وال مزع في أي‬.. ‫بما انھا بترح ركبتھا بشكل ممتاز‬
bursa ‫ فكري ب‬fluctuant mass ‫واھم اشي لما تشوفي‬ ‫الحالة عندھا االنتفاخ بركبتھا من‬
‫ فالتشخيص بطلع‬.. ‫قدام‬

3. ‫ لما عملوا صورة اشعة‬... ،‫ ھذا الوجع بروح لما يوخذ تروفين‬.. ‫ سنة عنده وجع بالفخذ فوق الركبة‬91 ‫مريض عمره‬
‫ شفنا‬,)Osteoid osteoma :small nidus (.8 mm

1. Dx of MM: bone marrow biopsy

2. What differentiates between disk prolapse and cauda equine syndrome: unilateral thigh

and buttock pain

3. Osteogenesis imperfect: type 1 collagen

4. Fracture of the neck of humerus: loss of sensation over the deltoid (axillary nerve injury)

5. Fracture of the tibial eminence, what structure would be affected? ACL

6. Not of the late complications of fracture: ostitis fibrosa cystica (or something like this)

7. Type of sternoclavicular joint: double plane joint

8. Osteoporosis: question from essential

9. Montigia fracture

10. All are true regarding fracture displacement except: location of proximal segment in

relation to the distal

11. Clubfoot (case)

12. Worst prognosis : neurotomesis

13. Septic arthritis : arthrocenthesis


14. Union of a simple uncomplicated transverse fracture of the tibia in an adult normally
takes

a. 6 weeks b. 8 weeks c. 12 weeks d. 18 weeks e. 26 weeks

‫ مثال مين العضة اللي في‬..‫ ببالنسبة لالناتومي حكا بجيب اشياء مميزة‬thigh ‫ اللي بغذيھا‬peroneal nerve

‫ العضلة اللي في‬forearm‫ بغذيھا‬ulnar ‫ اعرفو ال‬snuff box Cubital fossa, popletial fossa Innervation
for thenar and hypothenar muscles Carpat tunnel syndrom

You might also like